数学分析学习笔记

数学分析笔记

实数与序列

常见数集

下面给出一些集合的定义

\[\begin{aligned} \mathbb N &= \{0, 1, 2, \ldots\}\\ \mathbb Z &= \{\ldots,-1,0,1,\ldots\} \\ \mathbb Q& = \{\frac{p}{q}\vert p, q \in \mathbb Z, q\not = 0\} \end{aligned} \]

对于四则运算封闭的是 \(\mathbb Q, \mathbb R, \mathbb C\)

Dedekind分割

先定义一个分割

\(A, B\) 为有理数集合,满足:

  1. \(A, B \not = \emptyset, \mathbb Q = A \cup B, A \cap B = \emptyset\)
  2. \(\forall x\in A, y \in B, x < y\)

显然有四种情况

  1. \(A\) 中有最大值,\(B\) 中有最小值
  2. \(A\) 中无最大值,\(B\) 中有最小值
  3. \(A\) 中有最大值,\(B\) 中无最小值
  4. \(A\) 中无最大值,\(B\) 中无最小值

第一种情况不会出现,所以我们只讨论后面的三种。

Dedekind 分割就是 \(\{(A, B)是 \mathbb Q 的一个分割,且 A 中无最大值\}\)

每一个 Dedekind 分割唯一对应一个实数,无理数对应了第四种,有理数对应了第二种。

Dedekind 分割来定义无理数的运算

定义 \(x = (A_1, B_1), y = (A_2, B_2)\) ,若 \(A_1 \subset A_2\)\(x\le y\),若 \(A_1 \subset A_2\)\(A_1 \not = A_2\)\(x < y\)

性质

  1. 反称性:\(x\le y\)\(y\ge x\)
  2. 传递性:\(x\le y\)\(y \le Z\),则 \(x \le Z\)
  3. 完备性:\(x, y\in \mathbb R\),则 \(x\le y\)\(y\le x\)

四则运算定义

\(x + y = (A_3, B_3)\),其中 \(A_3 = \{r \in \mathbb Q\vert \exists r_1 \in A_1, \exists r_2 \in A_2, \text{s.t.} r = r_1 + r_2\}\)\(B_3 =\mathbb Q/A_3\)

负元 \(-x = (A_3, B_3)\),其中 \(A_3 = \{r \in \mathbb Q\vert \exists r_1 \in B_1, r = -r_1, r_1 不是最小元\}\)\(B_3 = \mathbb Q/A_3\)

则减法定义为 \(x - y = x + (-y)\)

乘法:

\(xy = (A_3, B_3)\)

  1. \(x\ge 0, y \ge 0, A_3 = \{r \in \mathbb Q\vert r < 0 或 \exists r_1 \in A_1, \exists r_2 \in A_2, \text{s.t.} r_1 \ge 0, r_2 \ge 0, r = r_1r_2\}, B = \mathbb Q / A_3\)
  2. \(x\ge 0, y < 0,xy = -(x(-y))\)
  3. \(x < 0, y < 0, xy = (-x)(-y)\)

逆元 \(A_3 = \{r \in Q\vert r\le 0 或 \exists r_1 \in B_1 , r_1 不是 B_1最小元,r=\frac{1}{r_1}\}, B = \mathbb Q / A_3\)

实数域范围内:

  1. 交换律
  2. 结合律
  3. 单位元
  4. 逆元
  5. 分配律

相容性条件

  1. 加法保序性 \(x\le y\)\(x + z \le y + z\)
  2. 乘法保序性 \(x\le y, z \ge 0\), 则 \(xz\le yz\)

实数确界存在定理

定义 :\(X \subset \mathbb R,x\not = \emptyset\)

  1. \(C \in \mathbb R, \text{s.t.} \forall x \in X, x \le C\) 则称 \(C\)\(X\) 的一个上界,下界同理
  2. \(X\) 既有上界又有下界,那么称 \(X\) 是有界集
  3. \(a\)\(X\) 的一个上确界,若 \(\forall C\)\(X\) 的上界,\(C \ge a\),记 \(a = \sup X\) 上确界,下确界是 \(\inf X\)

定理:确界存在定理:有上界或者下界的非空子集,必有唯一的上确界或者下确界。

证明:非空集 \(X\) 有上界,定义集合 \(A_1 = \{ r \in \mathbb Q \vert \exists(A, B) \in X, \text{s.t.}r\in A\}\)\(x_1 = (A_1, B_1) \in R\) 并且 \(\forall (A, B) \in X\) 我们有 \(A\subset A_1\)\(x_1 = (A_1, B_1)\) 是上界,现在设 \(x_0 = (A_0, B_0)\)\(X\) 的任一上界,根据定义,可以知道 \(A_1 \subset A_0, x_1 \le x_0\) 因此 \(x_1\)\(X\) 的唯一上确界。

实数分割

定义

  1. \(A, B\)\(R\) 的子集
  2. \(A,B\not = \emptyset, A \cap B = \emptyset, A\cup B = \mathbb R\)
  3. \(\forall x\in A, y \in B, x < y\)

实数连续性定理: \(\mathbb R\) 的一个分割,对应唯一一个实数 \(\alpha\) 并且 \(\alpha\)\(A\) 的最大值或者是 \(B\) 的最小值。

由确界定理可知,\(A\) 有上确界 \(\alpha\)

  1. \(\alpha \in A\)
  2. \(\alpha \in B\)\(\alpha\)\(B\) 中最小元

连续性推出确界存在性

证明: \(x\subset \mathbb R, x \not = \emptyset\) 有上界 \(c\)\(x\) 有上确界。

分为两种情况讨论

  1. \(c\in X\)
  2. \(c\not \in X, D = \{\beta\vert \beta 是 X 的上界\}\)\(C = \mathbb R / D,\exists \alpha = (C, D)\) 要么是 \(C\) 的最大值,要么是 \(D\) 的最小值,在分成两种情况讨论。
    1. \(\alpha\)\(C\) 的最大元,\(X \subset C\),这个和 \(\alpha\not \in D\) 矛盾。
    2. \(\alpha\)\(D\) 的最小元,则有 \(\alpha\) 为上确界
例一

证明: \(x= (0, 1), \sup X = 1\)

\(\forall x \in X, x < \frac{1 + x}{2} < 1\) 于是 \(1\) 是上确界 \(\Box\)

例二

证明:\(\sqrt 2\) 的存在性

\(X = \{x \vert x ^ 2 < 2\}\) ,证明 \(\left(\sup X\right) ^ 2 = 1\) 即可,记其为 \(\gamma\)

分成两个步骤证明:

  1. \(\gamma ^ 2 \ge 2\) 利用反证法,若 \(\gamma ^ 2 < 2\) ,构造 \(w \in (0, 1), \left(w + \gamma\right) ^ 2 < 2\) 即可。\((\gamma + w) ^ 2 < \gamma ^ 2 + 3\gamma w\) ,令 \(w = \frac{2 - \gamma ^ 2}{3 \gamma}\)
  2. \(\gamma ^ 2 \le 2\),反证法,考虑构造 \(u \in (0, \gamma), (\gamma - u) ^ 2 > 2\) 即可,令 \(u = \frac{r ^2 - 2}{2r}\) 即可,\(\Box\)

推广:\(\exists \gamma\in \mathbb R, \gamma ^ n = 2\)

有限或者无限小数表示实数 \(\alpha\),令 \(\alpha = (A, B)\), 找到 \(a_0, a_1, \ldots , a_n\) 使得 \(a_0.a_1\ldots a_n\in A, a_0.a_1\ldots a_n +\frac{1}{10 ^ n}\in B\) 即可。一直这么找下去。

数列及其收敛性

\(\mathbb N \rightarrow \mathbb R, n\mapsto a_n\)

例:\(\{1, \frac{1}{2},\frac{1}{3},\ldots, \frac{1}{n}\}\)

定义

\(\{a_n\}\) 为实数列,称 \(a\in \mathbb R\)\(\{a_n\}\) 的极限,\(\forall \varepsilon > 0, \exists N > 0\),使得 \(n > N\) 时,\(\vert a_n - a\vert < \varepsilon\),并且称 \(\{a_n\}\) 是收敛数列, \(a\) 为其极限,记为 \(\lim \limits_{n\to \infty} a_n = a \Leftrightarrow a_n\rightarrow a\)

例一

\((1)\lim\limits _{n\rightarrow \infty} \frac{1}{n} = 0, (2),\lim \limits _{n \rightarrow\infty} \frac{n ^ 2}{2 n ^ 2+ 1} = \frac{1}{2} (3)\lim \limits _{n \rightarrow\infty} \frac{1}{q ^ n} = 0, (\vert q \vert > 1)\)

  1. \(\forall \varepsilon > 0, \exists N = \left[\frac{1}{\varepsilon}\right]\), 在 \(n > N\) 时,\(\vert \frac{1}{n} - 0\vert < \varepsilon\)\(\Box\)
  2. \(\vert \frac{n ^ 2}{2n ^ 2 + 1} - \frac{1}{2}\vert = \frac{1}{4n ^ 2+2}<\frac{1}{n}, N = \left[\frac{1}{\varepsilon}\right]\),当 \(n > N\) 时,\(\vert \frac{n ^ 2}{2 n ^ 2 + 1} - \frac{1}{2}\vert < \frac{1}{n} < \varepsilon\)\(\Box\)
  3. 不妨设 \(q > 1, q = 1 + \alpha, \forall \varepsilon > 0\),想要找到一个 \(N\) 使得 \(\frac{1}{(1 + \alpha) ^ n}<\frac{1}{n \alpha} < \varepsilon\),于是 \(N = \left[\frac{1}{\varepsilon\alpha}\right]\),在 \(n > N\) 时,有 \(\frac{1}{q ^ n} = \frac{1}{(1 + \alpha) ^ n}< \frac{1}{n\alpha}<\varepsilon\)\(\Box\)
例二

\(\lim\limits _{n\rightarrow \infty} \sqrt[n] {n} = 1\)

证明:\(\forall \varepsilon > 0\),想找到一个 \(N\) 使得 \({n} < \left(1 + \varepsilon \right) ^ n\),则 \(\frac{n(n - 1)}{2}\varepsilon^2 > n\) 故而,\(\varepsilon ^ 2 > \frac{2}{n - 1}\)\(N = \left[\frac{2}{\varepsilon ^ 2} + 1\right]\)\(n > N\) 时,\(\sqrt [n] {n}<1 + \varepsilon\)\(\Box\)

例三

\(\{(-1) ^ n\}\) 发散,这个是简单的,假设存在极限 \(a\ge 0\),则 \(|a_{2n +1} - a|=1 + a > \varepsilon\)

收敛数列的性质

收敛的有界性和唯一性

定义:\(\{a_n\},\exists M > 0,\text{s.t.} \vert a_n \vert < M, \forall n \in \mathbb N\),则称 \(\{a_n\}\) 是有界数列。

定理,收敛的数列有界,收敛数列的极限唯一。

证明:

\(\{a_n\}\) 收敛,\(\varepsilon = 1,\exists N, \text{s.t.} n > N,\vert a _n \vert < |a| + 1\),此时 \(M = \max\{a_1, \ldots, a_N,\vert a\vert + 1\}\)

\(\{a_n\}\) 收敛于 \(a, b\),不妨设 \(a < b\),取 \(\varepsilon = \frac{b - a}{2}\)\(\vert a - b\vert \le \vert a - a_n\vert + \vert a_n - b\vert < \varepsilon + \varepsilon < b - a\),矛盾,\(\Box\)

子列

\(\{b_n\}\)\(\{a_n\}\) 的子列,若严格单调递增数列 \(\{k_n\}\) 使得 \(b_n = a_{k_n}\)

定理:收敛数列的子列一定收敛。

数列的四则运算,设 \(\{a_n\},\{b_n\}\) 均收敛, 那么 \(\{\frac{a_n}{b_n}\} = \frac{a}{b}\),即数列四则运算之后的结果就极限四则运算的结果。

这些证明都是平凡的。

例一

证明 \(a_n\rightarrow a\)\(\lim \limits _{n\rightarrow \infty} \frac{a_1 +\ldots+a_n}{n} = a\)

首先只要令 \(b_n = a_n - a\) 那么只要证明 \(\frac{b_1 + \ldots + b_n}{n} = 0\) 即可,\(\forall \varepsilon,\exists N_1,n>N_1,\vert b_n \vert < \varepsilon,\exists N _2 \text{s.t.} \frac{b_1 + \ldots b_{n_1}}{ N_2} < \varepsilon\),在 \(n > \max\{N_1, N_2\}\) 时候 \(\vert\frac{b_1 + \ldots + b_n}{n} - 0\vert <\vert \frac{b_1 + \ldots + b_{N_1}}{n}\vert + \vert \frac{b_{N_1 + 1} + \ldots + b_n}{n}\vert < 2\varepsilon\)\(\Box\)

夹挤定理

\(\{a_n\},\{b_n\},\{c_n\}\)

  1. \(\exists N, \text{s.t.} n > N, a_n\le b_n \le c_n\)
  2. \(\lim\limits _{n \rightarrow \infty} = \alpha, \lim\limits_{n\rightarrow\infty}c_n = \alpha\),则 \(\lim \limits_{n \rightarrow \infty} b_n = \alpha\)

证明:\(\forall \varepsilon > 0,\exists n > N, a - \varepsilon < a_n,b_n < b + \varepsilon ,a - \varepsilon < c_n < a + \varepsilon,\Box\)

例一

\(a > 0\)\(\exists n \in \mathbb{R},\text{s.t.} n\ge a,\lim\limits _{n\rightarrow\infty} a ^ {\frac{1}{n}} = 1\) 提示 \(\lim \limits_{n\rightarrow\infty} n ^ {\frac{1}{n}} = 1\)

  1. \(a\ge 1, \exists n\in \mathbb{ N},\text{s.t.}n\ge a, 1\le a ^ {\frac{1}{n}} \le n ^ {\frac{1}{n}}\),有夹挤定理,此时收敛于 \(1\)
  2. \(a < 1, \frac{1}{a} > 1,\exists n,\text{s.t.}n >\frac{1}{a},\lim\limits_{n\rightarrow\infty}a ^ {\frac{1}{n}} = \frac{1}{\lim\limits_{n\rightarrow\infty}\frac{1}{a} ^ {\frac{1}{n}}}= 1\)
例二

\(a _n > 0,\forall n \in \mathbb {R},\lim\limits_{n\rightarrow \infty} a_n = a\)\(\lim\limits_{n \rightarrow\infty} \left(\prod\limits_{k = 1} ^ n a_k \right) ^ {\frac{1}{n}} = a\)

证明1:

  1. \(a = 0\) 时,\(0 \le\lim \left(\prod\limits_{k = 1} ^ n a_k \right) ^ {\frac{1}{n}}\le \frac{\sum\limits_{k = 1} ^ n a_k}{n}\),由夹挤定理 \(\lim\limits_{n \rightarrow\infty} \left(\prod\limits_{k = 1} ^ n a_k \right) ^ {\frac{1}{n}} = 0\)
  2. \(\frac{n}{\sum\limits_{i =1} ^ n \frac{1}{a _i}}\le\lim\limits_{n \rightarrow\infty} \left(\prod\limits_{k = 1} ^ n a_k \right) ^ {\frac{1}{n}} \le \frac{\sum\limits_{k = 1} ^ n a_k}{n}\) 由夹挤定理可得

证明2:

\(a > 0\),由极限定义 \(\forall \varepsilon > 0, \exists N \in \mathbb{N}, \text{s.t.} n > N, 0 \le a - \frac{\varepsilon}{2} < a_n < a + \frac{\varepsilon}{2}\)

\[\begin{aligned} \lim\limits_{n \rightarrow\infty} \left(\prod\limits_{k = 1} ^ n a_k \right) ^ {\frac{1}{n}}&\le& \left(\prod\limits_{k = 1} ^ N a_k\right) ^ {\frac{1}{n}}\left(a + \frac{\varepsilon}{2}\right)^{\frac{n - N}{n}} \\ &=&\left(\frac{\prod\limits_{k = 1} ^ N a_k}{\left(a + \frac{\varepsilon}{2}\right) ^ N}\right) ^ {\frac{1}{n}}\left(a + \frac{\varepsilon}{2}\right) \end{aligned} \]

由于 \(\lim\limits_{n\rightarrow \infty} a ^ {\frac{1}{n}} = 1\)

\(\lim \limits _{n\rightarrow\infty}\left(\frac{\prod\limits_{k = 1} ^ N a_k}{\left(a + \frac{\varepsilon}{2}\right) ^ N}\right) ^ {\frac{1}{n}}\left(a + \frac{\varepsilon}{2}\right) = a + \frac{\varepsilon}{2}\)

\(\exists N_1 \in \mathbb{N},\text{s.t.} n > N_1, \left(\frac{\prod\limits_{k = 1} ^ N a_k}{\left(a + \frac{\varepsilon}{2}\right) ^ N}\right) ^ {\frac{1}{n}}\left(a + \frac{\varepsilon}{2}\right) < a + \frac{\varepsilon}{2} + \frac{\varepsilon}{2}\)

\(\exists N_2 = \max(N, N_1),n > N_2,\lim\limits_{n \rightarrow\infty} \left(\prod\limits_{k = 1} ^ n a_k \right) ^ {\frac{1}{n}} < a + \varepsilon\)

减号部分是同理的。

\(a = 0\),上面不需要证明 \(a - \frac{\varepsilon}{2}\) 部分即可

\(\Box\)

例三

\(a_n = \frac{\sum\limits_{k = 1} ^ n k!}{n!}\)\(\lim \limits _{n\rightarrow \infty} a_n = 1\)

证明:

\(1\le a_n \le 1+\frac{(n-2) (n-2)! + (n-1)!}{n!}<1 + \frac{2}{n}\)

由夹挤定理可证明极限为 \(1\)

\(\Box\)

例四

求解 \(\lim \limits_{n\rightarrow \infty} \left( \sqrt[2]{n ^ 2 + n} - n\right) ^ {\frac{1}{n}}\)

因为 \(\frac{1}{3}< \left( \sqrt[2]{n ^ 2 + n} - n\right) ^ {\frac{1}{n}} < \frac{1}{2}\)

保序性

  • \(\{a_n\}\rightarrow a, b > a, \exists N, n > N, \text{s.t.} a_n < b\)
  • \(\{a_n\}\rightarrow a,\{b_n\}\rightarrow b, b > a, \exists N, n > N, \text{s.t.} a_n < b_n\)
  • \({a_n} \rightarrow a, b_n \rightarrow b, \exists N, \text{s.t.} n > N, a_n < b_n\)\(a\le b\)

数列极限的推广

\(\{a_n\},\forall A > 0, \exists N, \text{s.t.} n > N, a_n > A\)\(\lim\limits_{n\rightarrow\infty}a_n = +\infty\)

\(\vert\lim\limits_{n\rightarrow\infty}a_n\vert = +\infty\) 记为 \(\lim\limits_{n\rightarrow\infty}a_n = \infty\)

\(\lim\limits_{n\rightarrow\infty}a_n = \infty\), \(\lim\limits_{n\rightarrow\infty}b_n = b(b > 0)\), 则 \(\lim\limits_{n\rightarrow\infty}a_nb_n = \infty\)

单调数列

定义: \(\{a_n\}\),满足 \(a_n \le a_{n + 1}, \forall n \in \mathbb N\),则称其为单调递增数列,没有等于号就是严格单调递增数列)

定理:(weierstrass 单调收敛定理):有界单调数列一定收敛

证明:\(X = \{a_1, a_2, \ldots,a_n\}\) 由确界存在定理 \(X\) 有上确界 \(\alpha\)\(\forall \varepsilon > 0, \exists N, \text{s.t.s},\alpha - \varepsilon < a_N\), 则 \(n > N\) 时,\(\alpha - \varepsilon < a_N \le a_n \le \alpha \le \alpha + \varepsilon\)

\(\Box\)

该定理和确界定理等价。

例一

\(a_n = \frac{\sum\limits_{k = 1} ^ n k!}{n!},\lim\limits_{n\rightarrow\infty} a_n = 1\)

例二

\(a_n = \sum\limits_{k = 1} ^ n \frac{1}{k}\),考察 \(\{a_n\}\) 的收敛性

证明:考虑这么如下情况 \(\sum\limits_{k = 2 ^ {n - 1} + 1} ^ {2 ^ n}\frac{1}{k} > 2 ^ {n - 1} \times \frac{1}{2 ^ n} = \frac{1}{2}\),而 \(a_{2 ^ n} = \sum_{i = 1} ^ n\sum_{k = 2 ^ {i - 1} + 1 }^{2 ^ i} \frac{1}{k} >\frac{n}{2}\)
于是,\(\{a_n\}\) 不收敛

\(\Box\)

例三

考察数列 \(\{a_n\}\) 满足 \(a_1 = b, a_{n + 1} = (a _n - a) ^ 2 + a_n\)

试问满足什么条件能让 \(a\) 收敛

\(a_{n + 1} - a_n = (a_n - a) ^ 2 \ge 0\),所以 \(a_n\) 单调增,设极限存在为 \(\alpha\)

对于 \(a_{n + 1} = (a_n - a) ^ 2 + a_n\) 两边取极限,\(\alpha = (\alpha - a) ^ 2 + \alpha\Rightarrow \alpha = a\)

  1. \(a\ge b, a_2 = (a_1 - a) ^ 2 + b \le a\Rightarrow (a - b) ^ 2 \le a - b\Rightarrow a - 1 \le b\)
    假设 \(a_n \le a \leftrightarrow a - 1 \le a_{n - 1}\)

定理

设闭区间 \([a_n, b_n], n = 1, 2 ,\ldots\) 满足:

  1. \([a_1, b_1]\supset [a_2, b_2]\supset\ldots\supset[a_n, b_n]\supset\)
  2. \(\lim\limits_{n\rightarrow\infty}(b_n - a_n) = 0\)

\(\exists ! \gamma \in \mathbb R, \text{s.t.},\gamma \in \bigcap\limits_{n = 1} ^ {\infty} [a_n, b_n]\)

证明:

\(a_1\le a_2\le \ldots \le a_n \le \ldots < b_1\)

\(\{a_n\}\) 单调增有上界,同理 \({b_n}\) 单调减有下界,故都收敛,设 \(\lim\limits_{n\rightarrow\infty} a_n = \gamma\)

要证明 \(\lim\limits_{n\rightarrow \infty} b_n = \gamma\)

\(\forall \varepsilon > 0\),由 \(\lim\limits_{n\rightarrow \infty}a_n = \gamma\)\(\exists N_1, \text{s.t.} n > N_1, \vert a_n - \gamma\vert < \varepsilon\)

由于 \(\lim\limits_{n\rightarrow\infty}(b_n - a_n) = 0\)\(\exists N_2, \text{s.t.} n > N_2, \vert a_n - b_n\vert < \varepsilon\)

\(N = \max(N_1, N_2),\text{s.t.} n > N, \vert a_n - \gamma\vert < 2\varepsilon\)

注:\((0, \frac{1}{n}), n\ge 1, \bigcap\limits_{n = 1} ^ \infty \left(0, \frac{1}{n}\right) = \emptyset\)

自然对数底数 e

定义

\(e_n = \left(1 + \frac{1}{n}\right) ^ n\\S_n = 1 + 1 + \frac{1}{2!} + \frac{1}{3}!+\ldots\)

\(e_n = 1 \cdot (1 + \frac{1}{n}) ^ n \le \left(\frac{n + 2}{n + 1}\right)^{n +1} = e_{n + 1}\)

\(\{e_n\}\) 单调递增

\(\frac{1}{2} \frac{1}{2} e_n \le \left(\frac{\frac{1}{2}+\frac{1}{2}+n(\frac{1}{n}+1)}{n+2}\right) ^ {n + 2}=1\)

所以 \(e_n \le 4\),故 \(e_n\) 有上界

\(\text{i.e.} \lim\limits_{n\rightarrow\infty} \left(1 + \frac{1}{n}\right)^n = e\)

\(S_n < S_{n + 1} < 1 + 1 + (1 - \frac{1}{2})+\ldots+(\frac{1}{n - 1} - \frac{1}{n}) = 3-\frac{1}{n}<3\)

\(\{S_n\}\) 收敛,下面证明 \(S = e\)

\[\begin{aligned} e_n &= \left(1 + \frac{1}{n}\right) ^ {n} \\ &=\sum\limits_{k = 0} ^ n\binom{n}{k} \frac{1}{n ^ k}\\ &= \sum\limits_{k = 0} ^ n\frac{1}{k!} \frac{n!}{(n - k)!n ^ {k}} < S_n \end{aligned} \]

由极限保序性 \(e \le S\),下面证明 \(S \le e\)

由上式 \(n > l\) 时候,对于固定的 \(l\)

\[e_n > \sum\limits_{k = 0} ^ l\frac{1}{k!} \frac{n!}{(n - k)!n ^ {k}} \]

\(n\rightarrow + \infty,e\ge \sum\limits_{k = 0} ^ l\frac{1}{k!}\),于是 \(\text{i.e.} e \ge S_l\)\(l \rightarrow \infty\) 故有 \(e\ge S\)

\(\Box\)

我们令 \(e = \sum_{k = 0} ^ n \frac{1}{k!} +\frac{\alpha_n}{nn!}\),我们可以估计出 \(\alpha_n\) 的范围,显然有 \(\alpha_n > \frac{n}{n + 1}\),我们再证明:

\[\begin{aligned} 0&<s_{n + m} - s_n\\ &=\sum_{k = n + 1} ^ {n + m} \frac{1}{k!}\\ &=\frac{1}{(n + 1)!}\left[1+ \frac{1}{n + 2} + \ldots+\frac{1}{(n + 2)\ldots(n + m)}\right] \\ &<\frac{1}{(n + 1)!}[1 + \frac{1}{n + 1} + \left(\frac{1}{n + 1}\right) ^ 2+\ldots+\left(\frac{1}{n + 1}\right)^{m-1}]\\ &<\frac{1}{(n + 1)!} \cdot\frac{1}{1-\frac{1}{n + 1}} = \frac{1}{n!n} \end{aligned} \]

再令 \(m\rightarrow \infty\),我们可以得到 \(0<e - s_n < \frac{1}{n!n}\)

而与 \(\frac{1}{q + 1} < \frac{\alpha_q}{q} < \frac{1}{q}\) 矛盾。

例一

\(\lim\limits _{n\rightarrow\infty} \frac{n}{(n!) ^ \frac{1}{n}} = e\)

解:设 \(a_n = \frac{n ^ n}{n!}\)\(\frac{a_{n + 1}}{a_n} = (\frac{n + 1}{n}) ^ {\frac{1}{n}}\)

所以 \(\lim \limits_{n\rightarrow \infty} \frac{a_{n + 1}}{a_n} = e\),则由夹挤定理的例二,有 \(\lim\limits_{n\rightarrow\infty} a_n ^ {\frac{1}{n}} =e\)

例二

求解

\[\begin{aligned} &=\lim\limits_{n\rightarrow \infty} \left(1 - \frac{1}{n}\right) ^ n \\ &=\lim\limits_{n\rightarrow \infty} \left(\frac{1}{\frac{n}{n - 1}}\right) ^ n \\ &=\frac{1}{\left(1 + \frac{1}{n - 1}\right) ^ {n -1}\left(1 +\frac{1}{n - 1}\right)} \\ &=e ^ {-1} \end{aligned} \]

例三

求解

\[\begin{aligned} \left(1 + \frac{k}{n}\right) ^ n \le \left(1+\frac{k}{n + 1}\right) ^ {n + 1} \end{aligned} \]

\[\begin{aligned} \left(1 + \frac{k}{n}\right) ^ n \le \left(1 + \frac{1}{n}\right) ^ {kn} < e ^ k \end{aligned} \]

有子列 \(\lim\limits_{n\rightarrow \infty} \left(1 + \frac{1}{n}\right) ^{nk} = e ^ k\)

因为其本身有界收敛,则 \(\lim\limits_{n\rightarrow \infty} \left(1 + \frac{k}{n}\right) ^ n = e ^ k\)

Cauthy 列与 Cauthy 收敛定理

Cauthy 列定义:设 \(\{a_n\}\) 是数列,\(\forall \varepsilon > 0, \exists N\),当 \(n, m > N\) 时,\(\vert a_m - a_n \vert < \varepsilon\)

定理: \(\{a_n\}\) 是收敛 \(\Leftrightarrow\) \(\{a_n\}\) 是Cauthy 列

例一

\(\{\frac{1}{n}\}\) 是 Cauthy 列

例二

\(a_n = \sum\limits_{k = 1} ^ n \frac{1}{k}\),他不是 Cauthy 列

有限覆盖定理

定义:\(X \subset \mathbb R, \{I_{\lambda}\}_{\lambda \in \Lambda}\) 为开集簇

\(X\) 满足 \(X \subset \bigcup\limits_{\lambda\in \Lambda}I_{\lambda}\)\(\{I_{\lambda}\}_{\lambda \in \Lambda}\)\(X\) 的开覆盖

\(\exists \lambda_1 , \ldots, \lambda_n,\text{s.t.} X \subset \bigcup\limits _{R = 1} ^n I_{\lambda _R}\)

定理:任何闭区间的开覆盖都有有限子覆盖

假设不存在区间 \(I_1 = [a, b]\) 不存在区间子覆盖。

那么,对于 \(\left[a, \frac{a + b}{2}\right]\) 或者是区间 \(\left[\frac{a + b}{2}, b\right]\) 肯定有一个区间不能被有限覆盖,设其为 \(I_2\),那么一定有一串区间

\[I_1 \supset I_2\supset \ldots \supset I_n \]

其中 \(\vert I_n \vert = \frac{1}{2 ^ n}(b - a)\)

那么通过闭区间套定理,得到 \(\exists\gamma \in \bigcap\limits_{i = 1} ^ {\infty} I_n\)\(\gamma \in [a, b] \subset S,\exists I _{\lambda_0} = (\alpha, \beta),\text{s.t.} \gamma \in (\alpha, \beta)\)

\(\delta = \min\{\gamma - \alpha, \beta - \gamma\}\)\(\exists m, \gamma \in I_m,\vert I_m \vert = \frac{1}{2 ^ m}(b - a) < \frac{1}{2}\gamma\),故存在一个区间包含 \(I_m\),矛盾,故有限覆盖定理得证。

Cauthy收敛定理

数列 \(\{a_n\}\) 收敛 \(\Leftrightarrow\) 数列 \(\{a_n\}\) 为 Cauthy 列

紧致集的定义:任何开覆盖都有有限子覆盖的集合称为紧致集

极限点的定义:称 \(\alpha \in \mathbb R\)\(X\subset \mathbb R\) 的一个极限点,如果 \(\forall \delta > 0\),在 \(B_\delta\left(\alpha\right) = (\alpha - \delta, \alpha + \delta)\) 中有 \(X\) 的无穷多项。在 \(B_\delta (\check{\alpha}) = B_\delta(\alpha)/\{\alpha\}\) 为去心邻域。

BW(Bolzano-Weierstrass)定理

定义:任何有界数列都有收敛子列

\(\{a_n\}\) 是有界数列,\(X = \{a_1, \ldots, a_n,\ldots\}\),由于 \(\{a_n\}\) 有界,则 \(\exists [a, b],\text{s.t.},X\subset [a,b]\)

claim: \([a, b]\) 存在 \(X\) 的极限点

反证法:\(\forall x \in [a, b], \exists \delta(x), \text{s.t.} B_{\delta(x)}\) 只包含 \(X\)\(\bigcup_{x\in[a,b]} B_{\delta(x)}\)\((a, b)\) 开覆盖,由有限覆盖定理可知,\(\exists [a, b]\) 的有限子覆盖 \(B_{\delta(x_1)},B_{\delta(x_2)},\ldots, B_{\delta(x_n)}\),由于其含有有限多个 \(X\) 中的点,但是 \(X\) 包含无穷多个点,矛盾。

\(x\in [a, b]\)\(X\) 的极限点,\(\forall n, \exists a_{k_n}\in X,\text{s.t.}, a_{k_n} \in B_{\frac{1}{n}}(\check{x})\),且 \(k_n\) 单调递增,知道 \(a_{k_n}\) 收敛

Cauthy 收敛定理的证明

\(\Rightarrow\)\(\{a_n\}\) 收敛,则 \(\forall \varepsilon > 0, \exists N, \text{s.t.} n > N, \vert a - a_n\vert<\varepsilon\),则 \(n, m>N, \vert a_n - a_m\vert < 2 \varepsilon\) 证明完毕。

\(\Leftarrow\),设 Cauthy列 \(\{a_n\}\)\(\forall \varepsilon > 0, \exists N, \text{s.t.} \vert a_n - a_m\vert < \frac{\varepsilon}{2}\)

显然 Cauthy 列是有界数列,根据 \(B-W\) 定理必然存在收敛子列,\(\vert a_{k_n} - a\vert < \frac{\varepsilon}{2},k_n > N,\vert a_n - a\vert < \vert a_n - a_{k_n}\vert + \vert a_{k_n} - a\vert < \varepsilon\)

则收敛,\(\Box\)

无穷级数

\(\{a_n\}\) 为数列,称 \(\sum\limits _{ n = 1} ^{\infty} a_n\) 为无穷级数,并且称 \(S_n = \sum\limits_{k = 1} ^ na_k\) 为无穷级数的部分和,如果 \(\{S_n\}\) 收敛至 \(S\),则称无穷级数收敛,记作 \(S = \sum\limits_{n = 1} ^ \infty a_n\)

若无穷级数收敛,\(\forall \varepsilon >0, \exists N, \text{s.t.}n> N, \vert \sum\limits_{k = n} ^ m a_k \vert<\varepsilon,\forall m \ge n\),这是柯西收敛准则。

若存在正项级数 \(\sum\limits_{n = 1} ^ \infty M_n\) 满足 \(\vert a_n \vert\le M_n\),若 \(M\) 收敛,那么 \(a\) 收敛

例一

\(\sum\limits_{n = 1} ^ \infty\frac{1}{n}\) 发散

例二

\(\sum\limits_{n = 1} ^ \infty \frac{1}{n ^2}\) 收敛

\(\frac{1}{n ^ 2}<\frac{1}{n(n - 1)}\),则原数列单调,并且有界,则收敛。

数列的上极限和下极限

例一

\(\{a_n\},a_n = (-1) ^ n + \frac{(-1) ^ n}{n}\)

\(a_{2n}\rightarrow 1, a_{2n - 1} \rightarrow -1\)

例二

\(\{\sin \frac{n\pi}{4}\},n\in \mathbb N\)

例三

\(\{1, 2, 1, 3, 1, 4, 1, 5,\ldots\}\)

\(a_{2n - 1} = 1, a_{2n} = n + 1\)

子列也有极限

定义,给定 \(\{a_n\},\mathbb R_{\infty} = \mathbb R \cup \{\pm \infty\}\)

\(E = \{x \in \mathbb R_{\infty}\vert 数列\{a_n\} 中存在子列 ,\text{s.t.}\lim\limits_{n\rightarrow \infty} a_{k_n} = x\}\)

\(E\) 中元素是 \(\{a_n\}\) 的部分极限

\(a_{*} = \inf E, a^{*} = \sup E\)

分别是 \(\{a_n\}\) 的上极限或者下极限

\(a_* = \lim\limits_{n\rightarrow \infty}\inf a_n\)

\(a^* = \lim\limits_{n\rightarrow\infty} \sup a_n\)

定理 : \(a_* \in E, a^*\in E\)

只需要证明 \(a_* \in E\)

  1. \(a_* = -\infty\) 此时 \(E\) 没有下界,若存在 \(M\) \(\text{s.t.} E \in [M, +\infty]\),从而 \(E \ge M\) 此时于无下界矛盾,则 \(E\) 无下界,\(\forall A,\exists a_{kn},\text{s.t.} a_{k_n} < -A\),则 \(\lim _{a_{k_n}} = -\infty\)
  2. \(a_* \in \mathbb R,\forall n, \exists x_n \in E, x_n \in [a_*, a_* + \frac{1}{n}), \exists\{a_n\}\) 的子列 \(\text{s.t.}a_{k_n}, a_{k_{n}}\),从而 \(\lim \limits_{n\rightarrow\infty}a_{k_n} = a_*\)
  3. \(a_* = +\infty\)\(\lim \limits_{n\rightarrow\infty}a_n = +\infty\),事实上,如果 \(\exists M > 0, \exists\) 子列 \(\{a_{k_n}\} \le M\)\({a_{k_n}}\) 存在子列 \(\{a_{k_{n_j}}\}\) 收敛到 \(\gamma \in (\infty, M]\),与 \(a_{*} = +\infty\)

\(\Box\)

注: 若 \(a_* \in \mathbb R\)

  1. \(\forall \varepsilon > 0, \exists N, \text{s.t.} n > N, a_* - \varepsilon \le a_n\)
  2. \(\forall \varepsilon > 0, \exists \{a_{k_n}\}, N, k_n > N, \text{s.t.} a_{k_n}\le a_* + \varepsilon\)

\(a^*\) 是同理的

\(\sup _n = \sup\{a_k\vert k \ge n\}\) \(\inf\) 同理

定理 \(\lim \limits_{n\rightarrow \infty}{\inf a_n} = a_*\)

  1. \(a_* = \infty\),由于存在子列 \(\{a_{k_n}\}\) 趋于 \(-\infty\),由于 \(\inf a_{n}\le \inf a_{k_n}\),则 \(\lim\limits_{n\rightarrow\infty} a_{k_n} = -\infty\)\(\lim\limits_{n\Rightarrow \infty} a_n = -\infty\)
  2. \(a_* \in \mathbb R, \forall \varepsilon > 0,\exists N , \text{s.t.} a_n \ge a_{*} - \varepsilon,\forall n, \exists \{a_{k_n}\}, k_n > n, a_{k_n} \le a_{*} + \varepsilon\)\(\inf a_n\in (a _* - \varepsilon, a_* + \varepsilon)\),从而 \(\lim\limits_{n\rightarrow\infty} \inf a_n = a_*\)
  3. \(a_* = +\infty\)\(\lim \limits_{n\rightarrow\infty}a_n = +\infty\),\(\forall M > 0, \exists N, \text{s.t.} n > N, a_n > M,\inf a_n \ge M\Rightarrow \lim\limits_{n\rightarrow \infty} \inf a_n = +\infty = a_*\)

定理:

  1. \(\lim\limits_{n\rightarrow \infty}\inf a_n \le \lim\limits_{n\rightarrow \infty} \sup a_n\)
  2. \(\lim\limits_{n\rightarrow \infty} a_n =a\Rightarrow\lim\limits_{n\rightarrow \infty}\inf a_n = \lim\limits_{n\rightarrow \infty} \sup a_n = a\)
  3. \(\exists N, \text{s.t.} n > N, a_n \le b_n, \lim\limits_{n\rightarrow \infty} \inf a_n \le \lim\limits_{n\rightarrow \infty} \inf b_n, \lim\limits_{n\rightarrow \infty} \sup a_n \le \lim\limits_{n\rightarrow \infty}\sup b_n\)
  4. \(\lim\limits_{n\rightarrow \infty} \inf a_n + \lim\limits_{n\rightarrow \infty} \inf b_n \le \lim\limits_{n\rightarrow \infty}\inf(a_n + b_n) \le \lim\limits_{n\rightarrow \infty} \inf a_n + \lim\limits_{n\rightarrow \infty} \sup b_n\)
  5. \(a_n, b_n \ge 0, \lim\limits_{n\rightarrow \infty} \inf a_n \cdot \lim\limits_{n\rightarrow \infty} \inf b_n \le \lim\limits_{n\rightarrow \infty} \inf(a_n b_n) \le \lim\limits_{n\rightarrow \infty} \inf a_n \cdot \lim\limits_{n\rightarrow \infty} \sup b_n\)

命题:

  1. \(\lim\limits_{n\rightarrow \infty} \inf(-a_n) = \lim\limits_{n\rightarrow \infty}\sup a_n\)
  2. \(a_n > 0, n\in \mathbb N, \lim\limits_{n\rightarrow \infty}\inf\frac{1}{a_n} = \frac{1}{\lim\limits_{n\rightarrow \infty} \sup a_n}\)
  3. \(\lim\limits_{n\rightarrow \infty} a_n = a,\lim\limits_{n\rightarrow \infty}\inf(a_n + b_n) = a + \lim\limits_{n\rightarrow \infty}\inf b_n\)\(\sup\) 同理,证明是简单的

例子:

例一

\(\{x_n\},\{y_n\}\) 满足

  1. \(\lim\limits_{n\rightarrow \infty} y_n = y\)
  2. \(y_n = x_n + 2x _{n + 1}\)

证明 \(x_n\) 收敛,并且求 \(\lim\limits_{n\rightarrow \infty} x_n\)

等式两边同时取上极限,

\(y - \lim\limits_{n\rightarrow \infty} \inf x_n = 2\lim\limits_{n\rightarrow \infty}\sup x_n\)

\(y - \lim\limits_{n\rightarrow \infty} \sup x_n = 2\lim\limits_{n\rightarrow \infty}\inf x_n\)

如果 \(x_n\) 有界,\(\lim\limits_{n\rightarrow \infty} \inf x_n = \lim\limits_{n\rightarrow \infty}\sup x_n = \frac{1}{3}y\),下面证明 \(x_n\) 有界, \(\exists M > 0, \text{s.t.}\vert x_1\vert\le M, \vert y_n\vert\le M\)\(\vert x_{n+ 1}\vert\le = \frac{1}{2}\vert y_n - x_{n}\vert \le M\) 则数学归纳法,于是 \(\{x_n\}\) 有界

例二

\(\{a_n\}\) 是基本列,则收敛

证明:

\(\forall \varepsilon > 0, \exists N > 0, \text{s.t.}n, m > N, \vert a_n - a_m\vert < \varepsilon\)\(m = N + 1\)\(a_{N + 1} - \varepsilon < a_n < a_{N + 1} + \varepsilon\)\(a_{N + 1} - \varepsilon < \lim\limits_{n\rightarrow \infty} \inf a_n \le \lim\limits_{n\rightarrow \infty}\sup a_n \le a_{N + 1} + \varepsilon\)

从而 \(0\le \lim\limits_{n\rightarrow \infty} \sup a_n - \lim\limits_{n\rightarrow \infty}\inf a_n \le 2 \varepsilon\) 由于 \(\varepsilon\) 的任意性,有 \(\lim\limits_{n\rightarrow \infty} \inf a_n = \lim\limits_{n\rightarrow \infty} \sup a_n\)

\(\{a_n\}\) 收敛

\(\Box\)

例三

\(b_1 = 1\)\(\{b_n\}\) 满足 \(b_{n + 1} = 1 + \frac{1}{b_n},n\ge 1\)

证明:

显然 \(b_n\in [1, 2]\)\(\alpha = \lim\limits_{n\rightarrow \infty} \inf b_n, \beta = \lim\limits_{n\rightarrow \infty} \sup b_n\)

那么有 \(\alpha = 1 + \frac{1}{\beta},\beta = 1 + \frac{1}{\alpha}\),那么 \(\alpha = \beta = \frac{1 + \sqrt{5}}{2}\)

例四
  1. \(\lim\limits_{n\rightarrow \infty} a_n = a\)
  2. \(\lim\limits_{n\rightarrow \infty} \left(b_n ^ 2 - a_nb _n - 6 a_n ^ 2\right) = 0\)

让你证明 \(\lim\limits_{n\rightarrow \infty} \sup b_n \le 3 a_n\)

证明:

\(\lim\limits_{n\rightarrow \infty} \left(b_n - \frac{1}{2}a_n\right) ^ 2 = \frac{25}{4} a ^ 2\),从而 \(\lim\limits_{n\rightarrow \infty} \sup \left(b_n - \frac{1}{2} a_n\right) \le \frac{5}{2} a_n\)

然后证明完了

例五

\(\{a_n\},a_n > 0\)

\(\lim\limits_{n\rightarrow \infty} \inf \frac{a_{n + 1}}{a_n} \le \lim\limits_{n\rightarrow \infty} \inf a _n ^ {\frac{1}{n}} \le \lim\limits_{n\rightarrow \infty} \sup a_n ^ {\frac{1}{n}}\le \lim\limits_{n\rightarrow \infty} \sup \frac{a_{n + 1}}{a_n}\)

证明:

\(\lim\limits_{n\rightarrow \infty} \sup \frac{a_{n + 1}}{a_n} \in \mathbb R,\lim\limits_{n\rightarrow \infty} \inf \frac{a_{n + 1}}{a_n} = \alpha\),则 \(\forall \varepsilon 0, \exists N \in \mathbb N,\text{s.t.} n \ge N, \frac{a_{n + 1}}{a_n} > \alpha - \varepsilon\),则 \(a_n \ge a_{n - 1} (\alpha - \varepsilon) \ge \ldots \ge a_{N} (\alpha - \varepsilon)^{n - N}\)

\(\text{i.e.} a_n ^ {\frac{1}{n}} >\left[\frac{a_N}{(\alpha - \varepsilon) ^ N}\right] ^ {\frac{1}{n}}(\alpha - \varepsilon)\),则 \(\lim\limits_{n\rightarrow \infty} \inf a_{n} ^ {\frac{1}{n}}\ge (\alpha - \varepsilon) \lim\limits_{n\rightarrow \infty}\left(\frac{a_N}{\alpha - \varepsilon}\right) ^ {\frac{1}{n}} = \alpha - \varepsilon\)\(\varepsilon\) 的任意性,可以知道 \(\lim\limits_{n\rightarrow \infty} \inf a_{n} ^ {\frac{1}{n}} \ge \alpha\),上极限是类似的。 \(\Box\)

Stolz-Cesaro 定理

定理:\(\{a_n\},\{b_n\}\) 满足
1

  1. \({b_n}\) 严格单调递增
  2. \(\lim\limits_{n\rightarrow \infty} b_n = + \infty\)
  3. \(\lim\limits_{n\rightarrow \infty} \frac{a_n - a_{n - 1}}{b_n - b_{n - 1}} = A\)

\(\lim\limits_{n\rightarrow \infty} \frac{a_n}{b_n}= A\)
注:此处 \(A\) 可以为 \(\pm \infty\)

证明:

分三种情况讨论
1:\(A\in \mathbb R,\forall \varepsilon > 0 , \exists N, \text{s.t.} n > N, A - \varepsilon < \frac{a_n - a_{n - 1}}{b_n - b_{n - 1}} < A + \varepsilon\)

从而有取 \(k = N + 1, \ldots, n\)

\((b_k - b_{k - 1})(A - \varepsilon) < a_{k} - a_{k - 1} < (A + \varepsilon)(b_k - b_{k - 1})\)

累加

从而有 \(\left(1 - \frac{b_N}{b_n}\right)(A - \varepsilon) < \frac{a_n}{b_n} - \frac{a_N}{b_n} < \left(1 - \frac{b_N}{b_n}\right)(A + \varepsilon)\)\(n\rightarrow \infty\),有 \(A - \varepsilon\le \lim\limits_{n\rightarrow \infty} \inf\frac{a_n}{b_n}\le \lim\limits_{n\rightarrow \infty} \sup \frac{a_n}{b_n }\le A + \varepsilon\) 由于 \(\varepsilon\) 的任意性,\(\lim\limits_{n\rightarrow \infty} \frac{a_n}{b_n} = A\)

2:\(A = +\infty, \exists N, \text{s.t.} n > N, a_n - a_{n - 1} > b_n - b_{n - 1} > 0\Rightarrow a_n > a_{n - 1}\)\(\lim\limits_{n\rightarrow \infty} a_n = +\infty\)

\(\lim\limits_{n\rightarrow \infty} \frac{b_n - b_{n - 1}}{a_n - a_{n - 1}} = 0\) 从而有 \(\lim\limits_{n\rightarrow \infty} \frac{b_n}{a_n} = 0\),所以 \(\lim\limits_{n\rightarrow \infty} \frac{a_n}{b_n} = +\infty = A\)

3:极限为负无穷的时候,由前两种可知。

逆命题不一定成立!!!

定理:

\(\{a_n\}, \{b_n\}\) 满足

  1. \(\{b_n\}\) 严格单调
  2. \(\lim\limits_{n\rightarrow \infty} a_n = 0, \lim\limits_{n\rightarrow \infty}b_n = 0\)
  3. \(\lim\limits_{n\rightarrow \infty}\frac{a_n - a_n - 1}{b_n - b_n - 1} = A\)

\(\lim\limits_{n\rightarrow \infty} \frac{a_n}{b_n} = A\)

证明:

\(A = \pm\infty\) 留做练习

\(A\in \mathbb R\) 时, 当 \(n, m>N\) 时候,\(\vert\frac{a_n - a_m}{b_n - b_m}\vert<\varepsilon\),令 \(m\rightarrow \infty\)\(\vert \frac{a_n}{b_n} - A\vert < \varepsilon\) \(\Box\)

例一

给定数列 \(\{a_n\}, \lim\limits_{n\rightarrow \infty} a_n = 0\),则 \(\lim\limits_{n\rightarrow \infty} \frac{a_1 + \ldots+a_n}{n} = a\)

一下证明完了。

引理:

\(s_1,\ldots,s_n, t_1, \ldots,t_n\),令 \(S_k = \sum_{i = 1} ^ k s_i\) 则有 \(\sum\limits_{k = 1} ^ n s_k t_k =S_n t_n - \sum\limits_{k = 1} ^ {n - 1}{S_k (t_{k + 1} - t_k)}\)

证明是显然的。

例二

\(\lim\limits_{n\rightarrow \infty} \frac{a_1 + a_2 + \ldots + a_n}{n} = \alpha\),,证明 \(\lim\limits_{n\rightarrow \infty} \frac{1}{\ln n}\sum_{k = 1} ^ n \frac{a_k}{k} = \alpha\)

\(S_k = a_1 + \ldots +a_k, S_0 = 0\)

\(\begin{aligned}\lim\limits_{n\rightarrow \infty} \frac{1}{\ln n}\sum_{k = 1} ^ n \frac{a_k}{k} &\overset{\text{Abel}}{=}\lim\limits_{n\rightarrow \infty} \frac{1}{\ln n}\left(\frac{S_n}{n}-\sum\limits_{k = 1} ^ {n - 1} S_k \left(\frac{1}{k + 1} - \frac{1}{k}\right)\right)\\&\overset{}{=} \lim\limits_{n\rightarrow \infty} \frac{1}{\ln n}\left(\frac{S_n}{n} +\sum\limits_{k = 1} ^ {n - 1}\frac{S_k/k}{k + 1}\right)\\ &=\lim\limits_{n\rightarrow \infty} \frac{1}{\ln n}\sum\limits_{k = 1} ^ {n - 1} \frac{S_k/k}{k + 1}\\&\overset{S-C}{=} \lim\limits_{n\rightarrow \infty} \frac{\frac{S_{n - 1}}{n(n - 1)}}{\ln n - \ln(n - 1)} \\&=\lim\limits_{n\rightarrow \infty} \frac{S_{n - 1}}{n - 1}\frac{1}{n(\ln n - \ln (n - 1))}\\&=\lim\limits_{n\rightarrow \infty} \frac{1}{\ln \left(\frac{n}{n - 1}\right) ^ n} \\&\overset{承认}{=} \frac{1}{\ln\lim\limits_{n\rightarrow \infty}\left(1 + \frac{1}{n - 1}\right) ^ {n}}\\ &= \frac{1}{\ln e}\\ &= 1\end{aligned}\)

实数完备性六大定理:

  1. 确界存在原理
  2. Weierstrass 单调收敛定理
  3. Cauthy-Canter 闭区间套定理
  4. Heine-Borel 有限覆盖定理
  5. Bolzano-Weierstrass 列紧性定理
  6. Cauthy 收敛定理

函数的连续性

集合的映射

集合:\(X\)\(x\) 属于 \(X\),记作 \(x\in X\)

映射:存在集合 \(X, Y\),存在某种法则 \(f\)\(\text{s.t.}\forall X, \exists! y \in Y\) 与之对应,称 \(f\)\(X\)\(Y\) 的映射,记 \(y = f(x)\),记 \(f:X\rightarrow Y, x\mapsto y\),称 \(X\) 为映射的定义域,\(y\) 为目标域,记 \(f(x) = \{ f(x)\vert \forall x \in X \}\subset Y\),称 \(f(x)\) 是映射的值域。

例一

\(f:\mathbb N \rightarrow \mathbb Z, n\mapsto f(n) = 2n - 5\)

例二

\(\mathbb R ^ 2 = \mathbb R \times \mathbb R,(x, t) \mapsto f(x, t) = (x - t\gamma, t)\) 称为从一个惯性系 \((x, t)\) 到另外一个惯性系 \((x',t')\) 的伽利略变换

定义:给定集合 \(X, Y\),映射 \(f:X\rightarrow Y\)

  1. \(f\) 为单射,若 \(\forall x_1, x_2 \in X, x_1 \not = x_2, f(x_1)\not = f(x_2)\)
  2. \(f\) 为满射,若 \(\forall y \in Y, \exists x, \text{s.t.} f(x) = y\)
  3. \(f\) 既是单射又是满射,则是双射。

\(f:X\rightarrow Y\) 是单射,可以定义 \(g:f(x)\subset Y \rightarrow X\),满足 \(f(g(y)) = y, \forall y \in f(x)\),称 \(g\)\(f\) 逆映射,记为 \(f^{-1}\)

复合映射

存在 \(f:X\rightarrow Y,g:Y\rightarrow Z\) 定义 \(h,\text{s.t.} \forall x\in X, h(x) = g(f(x))\),称 \(h\)\(f\)\(g\) 的复合映射,记为 \(g\circ f\)

例:\(f:\mathbb R ^ 2 \rightarrow \mathbb R ^ 2, v\in \mathbb R\\(x, t)\mapsto(x',t') = (x - vt, t),g:\mathbb R ^ 2 \rightarrow R \\(x, t) \mapsto \sqrt{x ^ 2 + t ^ 2}\)

\((g\circ f)(x, t) = \sqrt{(x - vt)} ^ 2 + t ^ 2\)

定义:若 \(f:X \rightarrow Y\) 是双射,则 \(f^{-1}\),满足 \((f^{-1}\circ f)(x) = \text{id}_X,(f \circ f ^ {-1})= \text{id}_{Y}\)

\(f \circ f ^ {-1}\)\(X\) 上的恒同映射,称为 \(\text{id}_X\),对于 \(f:X\rightarrow Y, D\subset Y\),记 \(f ^ {-1} (D) = \{x\in X\vert f(x)\in D\}\)

集合的势

给定一个集合 \(X, Y\),若存在双射 \(f:X\rightarrow Y\),那么称 \(X\)\(Y\) 是等势的,记作 \(X \sim Y\),等势是一种等价关系。

  1. 反身性:\(X\sim X\)
  2. 对称性,若 \(X\sim Y\),那么 \(Y\sim X\)
  3. 传递性 \(X\sim Y, Y \sim Z\),那么 \(X \sim Z\)

格局上述的等价关系,每个集合在一个等价类里面,集合 \(X\) 在的等价类成为集合的势或者是基数,记为 \(\text{Card} X\)

\(X\)\(Y\) 的一个子集等价,那么称 \(X\) 的基数小于等于 \(Y\) 的基数,记为 \(\text{Card} X\le \text{Card} Y\)

\(\text{Card} X\le \text{Card} Y\)\(\text{Card} X\not = \text{Card} Y\),则 \(\text{Card} X< \text{Card} Y\)

例一

\(X = \mathbb N, Y = \{2n \vert n\in X\}, f:X\rightarrow Y, n\mapsto f(x) =2n\)

例二

\(f:(-1, 1) \rightarrow \mathbb R, x\mapsto f(x) = \frac{x}{1 - \vert x \vert}\),从而 \(\text{Card}(-1, 1) = \text{Card}(\mathbb R)\)

有限集,可数集和不可数集

定义:对于集合 \(X\)

  1. \(\exists n, \text{s.t.} X \sim \{1, \ldots, n\}\),称 \(x\) 为有限集
  2. \(X\sim \mathbb N\)\(X\) 为可数集
  3. \(X\) 不是上面两个,那么是不可数集
  4. \(X\) 是有限集或者是可数集,那么称其为至多可数集。
例一

\(\mathbb Z \sim \mathbb N\)

例二

\([0, 1]\)\((0, 1)\) 等势。

\(f:[0, 1] \rightarrow (0, 1)\\ f(x)=\begin{cases}\frac{1}{2} & x = 0\\ \frac{1}{n + 2} & x = \frac{1}{n}\\ x & x \not = \frac{1}{n}, x \in [0, 1], x \not = \frac{1}{n}, x\not = 0\end{cases}\)

定理:

  1. 可数集的无限子集仍是可数集。
  2. 至多个至多可数集的并仍然是可数集
  3. 有限个可数集的直积仍然是可数集

证明:

  1. \(A\) 是可数集,\(E\subset A\) 的无限子集,令 \(k_1\) 为最小正数 \(\text{s.t.} a_{k_1} \in E\),依次有 \(k_1, \ldots, k_{s - 1} = \{a_n\}\),令 \(k_s\) 为最小的大于 \(k_1, \ldots, k_{s - 1}\) 的正数使得 \(a_{k_s} \in E\) 给出集合 \(f:E\rightarrow\mathbb N, f(a_{k_n}) = n\) 为双射,所以 \(E\) 为可数集。
  2. \(E_n\) 为可数集,\(n = 1, 2, \ldots\),说明 \(\bigcup\limits_{n = 1} E_n\) 也是可数集

\(x_{11},x_{21},x_{12},x_{31},x_{22},x_{13},\ldots\) 然后剔除相同元素,取第一次出现的位置

定理 : \(\text{Card}\mathbb N<\text{Card}\mathbb R\)

证明:

反证法, \(f:\mathbb N \rightarrow [0, 1]\\ n \mapsto f(n) = x_n\) 的双射,将线段三等分,至少有一个区间不含有 \(x_n\),逐步这么找,得到一个闭区间套,然后得到 \(\gamma \not = x_1, x_2, \ldots,x_n\)

Schroder_Bernstein 定理:给定集合 \(A, B,\text{Card} A \le \text{Card} B, \text{Card} B\le \text{Card} A\),则 \(\text{Card} A = \text{Card} B\)

证明:

\(f:A \rightarrow B\) 为单射 \(g:B\rightarrow A\) 为单射,不妨设 \(g(B) \not = A\),令 \(A_0 = A\backslash g(B), A_1 = g(f(A_0)), A_k = g(f(A_{k-1}))\)

\(h(x) = \begin{cases}f(x) & x\in \bigcup\limits_{k = 0} A_k\\g^{-1}(x) & x\not\in \bigcup \limits_{k = 0} A_k\end{cases}\)

想说明 \(h:A\rightarrow B\) 是双射

  1. \(h\) 是单射,只需证明 \(x_1\not\in \bigcup \limits_{k = 0} A_k, x_2\not\in \bigcup \limits_{k = 0} A_k,g ^ {-1}(x_1) \not = f(x_2)\),使用反证法,如果存在 \(x_1, x_2\) 不符合此条件,那么 \(g ^ {-1}(x_1) = f(x_2)\),有 \(x_1 = g\circ f(x_2)\),由 \(x_2\not\in \bigcup \limits_{k = 0} A_k\), \(\exists m \ge 0, x _2\in A_m\),此时 \(x_1\in A_{m + 1}\) 矛盾!
  2. \(h\) 是满射,只需证明 \(g(y)\not\in \bigcup \limits_{k = 0} A_k, y\not\in f\left(\bigcup \limits_{k = 0} A_k\right)\) 即可,反证法,如果 \(g(y) \in \bigcup \limits_{k = 0} ^ \infty A_k, \exists m > 0, g(y) \in A_{m + 1}, \exists x\in A_m, g(y) = g(f(x))\),由于其是单射,所以 \(y = f(x)\),则 \(y = f(x) \in f\left(A_m\right)\subset f\left(\bigcup \limits_{k = 0} A_k\right)\),矛盾!

从而 \(h\) 是双射,所以 \(\text{Card} A=\text{Card} B\)

\(\Box\)

定义: \(X\) 为非空集合,由 \(X\) 的所有非空子集构成的集合为 \(X\) 的幂集,记为 \(P(x)\)

定义 \(\text{Card} X < \text{Card}P(X)\)

证明:

\(f:X\rightarrow P(X),x\mapsto \{x\}\),从而 \(\text{Card} X \le \text{Card} P(X)\),反证法,如果存在双射 \(g:X\rightarrow P(X)\),定义集合 \(Y = \{x \in X\vert x\not \in g(x)\},\exists x_0 \in X, \text{s.t.} g(x_0) = Y\)

  1. \(x_0 \in Y, x_0 \not \in g(x_0) = Y\) 矛盾
  2. \(x_0 \not \in Y, x_0 \in Y\) 矛盾

从而 \(\text{Card} X < \text{Card} P(X)\)

函数的概念

定义:设 \(X, Y \subset \mathbb R\)\(f\) 为从 \(X\)\(Y\)\(f\) 为从 \(X\)\(Y\) 的一元函数,记 \(y = f(x)\),下面出现的函数均为一元函数

\(x\) 称为自变量,\(y\) 称为因变量,称 \(X \times Y\) 的子集,\(\Gamma _f = \{(x, y)\vert y = f(x), \forall x \in X\}\) 为函数的图像。

定义,如果 \(f, g\) 均为从 \(X\)\(Y\) 的函数,那么可以定义

  1. \(\alpha, \beta \in \mathbb R, (\alpha f + \beta g)(x) = \alpha f(x) + \beta g(x), \forall x \in X\)
  2. \((f\cdot g) (x) = f(x) \cdot g(x), \forall x \in X\)
  3. \(g(x) = 0\)\(\left(\frac{f}{g}\right)(x) = \frac{f(x)}{g(x)}\)

若函数 \(f:X\rightarrow Y\) 为双射,则 \(f ^ {-1}\) 存在,称为 \(f\) 的逆函数

定义:设函数 \(f:X\rightarrow Y\),称 \(f\) 为单调递增函数,若 \(\forall x_1, x_2\in X,f(x_1)\le f(x_2)\),如果有严格就是没有等号的情况,对于单调递减同理。

定理:如果是严格单调递增/递减函数,那么逆函数存在,记为 \(f ^ {-1}\),且 \(f ^ {-1}\) 也是严格单调递增/递减函数。

证明:逆函数存在是显然的,那么设 \(f\) 是严格单调递增函数,那么要证明他的逆也是严格单调递增函数。

使用反证法,如果存在 \(y_1, y_2\in f(X)\),并且 \(y_1<y_2\),满足 \(f^{y_1}\ge f^{-1}(y2)\),由于 \(f\) 是双射,那么 \(f\circ f ^ {-1} (y_1) \ge f \circ f ^ {-1} y_2\)\(y_1 \ge y_2\),矛盾!

函数的极限

\(f\) 为定义在 \(x\subset \mathbb R\) 上的函数, \(a\in \mathbb R\) 的一个极限点,\(A\in \mathbb R\),若 \(\forall \varepsilon > 0, \exists \delta > 0, \text{s.t.} \forall x \in \mathring{U}_{X}(a, \delta) = \{x\in X\vert 0 < \vert x - a\vert < \delta\}\)

均有 \(\vert f(x) - A\vert < \varepsilon\),称函数 \(f\)\(a\) 点的极限为 \(A\),记为 \(\lim\limits_{x\rightarrow a} = A, f(x)\rightarrow A,x\rightarrow a, x\in \mathring{U}_{X} (a, \delta)\)

注:

  1. \(a\)\(X\) 的极限点,\(\forall \delta > 0, \mathring{U}_{X} (a, \delta) \not = \emptyset\)
  2. \(a\) 未必属于 \(X\)\(f\)\(a\) 点上未必有定义,即使 \(f\)\(a\) 上有定义,\(A\)\(f(a)\) 也未必相等

Heine 定理

\(f\) 为定义在 \(X\) 上的函数,\(a\)\(X\) 的极限点,\(\lim\limits_{x\rightarrow a} f(x) = A\) 的充分必要条件是 \(\forall \{x_n\} \subset X\backslash \{a\}\)\(\lim\limits_{n\rightarrow \infty} x_n = a, \lim\limits_{n\rightarrow \infty} f(x_n) = A\)

证明:

\(\Rightarrow,\forall \varepsilon > 0, \exists \delta > 0, \text{s.t.} \vert f(x) - A\vert <\varepsilon , \forall x \in \mathring{U}_{X} (a, \delta)\) 由于 \(\lim\limits_{n\rightarrow \infty} x_n = a, \exists N \in \mathbb N, \text{s.t.} n > N, x_n \in \mathring{U}_{X} (a, \delta)\)\(\vert f(x_n) -A\vert < \varepsilon,\text{i.e.} \lim\limits_{n\rightarrow\infty} f(x_n) = A\)

\(\Leftarrow,\)\(f\)\(a\) 点上极限不是 \(A\)\(\exists \varepsilon > 0, \forall n \in \mathbb N \exists x_n \in \mathring{U}_{X} \left(a, \frac{1}{n}\right), \vert f(x_n) - A\vert \ge \varepsilon\),由于 \(x_n\rightarrow a, f(x_n)\rightarrow A\) 矛盾!

\(\Box\)

Cauthy 收敛定理

定理:设 \(f\) 为定义在 \(X\) 上的函数,\(a\)\(X\) 的极限点,则

\(\lim\limits_{x\rightarrow a} f(x) = A\Leftrightarrow \forall \varepsilon > 0, \exists \delta > 0, \text{s.t.} x_1, x_2, \in \mathring{U}_X(a, \delta)\),有 \(\vert f(x_1) - f(x_2) \vert < \varepsilon\)

证明:

\(\Rightarrow , \forall \varepsilon > 0, \exists \delta > 0, \text{s.t.} \forall x \in \mathring{U}_X(a, \delta) \vert f(x) - A\vert < \frac{\varepsilon}{2}\)

\(\forall x_1, x_2\in \mathring{U}_X(a, \delta),\vert f(x_1) - f(x_2)\vert < \varepsilon\)

\(\Leftarrow\)\(\{x_n\} \subset X\backslash \{a\}\)\(\lim\limits_{n\rightarrow\infty} x_n = a\) 对于 \(\varepsilon > 0, \exists N, \text{s.t.}n, m > N, x_n, x_m \in \mathring{U} _x (a,\delta)\),由于假设可知 \(\vert f(x_n) - f(x_m)\vert < \varepsilon\),故 \(\{f(x_n)\}\) 是基本列,由于数列的Cauthy收敛订立,可以知道 \(\lim _{n\rightarrow \infty} f(x_n) = A\),设 \(\forall \{y_n\}\subset X \backslash \{a\}\)\(\lim_{n\rightarrow} y_n = a\)\(\lim\limits_{n\rightarrow\infty} f(y_n) = A\),故而任意的数列都收敛 \(A\)

\(\Box\)

极限的性质

  1. \(f\) 是定义在 \(X\) 上的函数 \(a\) 是极限点,所以 \(a\) 点极限唯一
  2. \(f\)\(a\) 点有极限,则 \(\exists \delta > 0, x \in \mathring{U}_X(a, \delta), f(x)\) 有界

定理:设 \(f, g\) 为定义上 \(X\) 上的函数,\(a\) 是极限点,若 \(\lim\limits_{x\rightarrow a}f(x) = A, \lim\limits_{x\rightarrow a} g(x) = A\)

则类比数列极限四则运算会有函数的极限的四则运算

定理:保序性:设 \(f, g, h\) 定义在 \(X\) 上的函数,\(a\)\(X\) 的极限点,且 \(\lim\limits_{x\rightarrow a} f(x) = A, \lim\limits_{x\rightarrow a} g(x) = B\)

  1. \(A < B\)\(\exists \delta > 0, \text{s.t.} \forall x \in \mathring{U}_X(a, \delta), f(x) < g(x)\)
  2. \(\exists \delta > 0, f(x) \le g(x),\forall x \in \mathring{U}_X (a, \delta)\)\(A \le B\)
  3. \(f(x) \le h(x),h(x)\le g(x), \exists \delta > 0, \forall \mathring{U}_X(a, \delta)\)\(A = B\),则 \(\lim\limits_{n\rightarrow a} h(x) = A\)

定理:复合函数的极限

\(f:X\rightarrow Y\)\(x_0\)\(X\) 的极限点

\(g:Y\rightarrow Z\), \(f(x) \subset Y\), \(y_0\)\(Y\) 的极限点

\(\lim\limits_{x\rightarrow x_0} f(x) = y_0\),且 \(\exists \delta > 0, \text{s.t.} \forall x \in \mathring{U}_X(x_0, \delta),f(x) \not = y_0,f(x)\not=y\)

\(\lim\limits_{x\rightarrow x_0}(g\circ f) (x) = A\)

例子:\(f(x) = 0\)\(g(y) = \begin{cases} 1 & y = 0\\0 &y \not = 0\end{cases}\)

左右极限

定义:\(f\) 定义在 \(X\) 上的极限,\(a\)\(X\) 的极限点,称 \(a\) 处存在左极限,若将 \(X \cap (-\infty, a)\)\(a\) 处有极限。

记作:\(\lim\limits_{x\rightarrow a^-} = A\)

右边的极限是同理的

定理:\(f\) 为定义在 \(X\) 上的函数,\(a\) 为极限点,则 \(f\)\(a\) 处极限存在 \(\Leftrightarrow\) \(f\)\(a\) 上有左右极限,且相等。

一个重要的极限

\(\lim\limits _{x\rightarrow 0} \frac{\sin x}{x} = 1\)

证明:

画一个形得到 \(\sin x < x < \tan x\),转化为 \(\frac{\sin x}{x}\) 的情况下,然后夹一下即可。

指数函数

给出 \(a ^ x(a > 1),x\in \mathbb R\),定义:

  1. \(x \in \mathbb N\),归纳方法定义,\(a ^ 0 = 1\)\(a ^ {n + 1} = a \times a ^ n\)
  2. \(n \in \mathbb Z\)\(n\in \mathbb N\)\(a ^ {-n} = \frac{1}{a ^ n}\)
  3. 由于实数完备性定理,\(\forall n \not in N, \exists !\gamma,\text{s.t.}\gamma ^ n = a, \text{s.t.}(a ^ {\frac{1}{n}}) ^ n = a\) , \(a ^ \frac{m}{n} = \left(a ^ {\frac{1}{n}}\right) ^ m\)
  4. \(x\in \mathbb R \backslash \mathbb Q\)\(x_0\) 为有理数的极限, \(\lim \limits_{x\rightarrow x_0 \in, x\in \mathbb Q} = a ^ {x_0}\) 由于 \(\lim\limits_{x \rightarrow 0 ^ +} = a ^ x = 0\)\(\forall \varepsilon > 0, \exists \delta > 0, \text{s.t.} \forall x \in \mathring{U} _Q (0, 2 \delta), \vert a ^ x - 1 \vert < \frac{\varepsilon}{ M}, M = \sup a ^ x, x \in \mathring{U}_Q(x_0, 1), \forall x_1, x_2 \in \mathring{U}_Q(x_0, \delta),\vert a ^ {x_1 } - a ^ {x_1}\vert = a ^ {x_1}\vert 1 - a ^ {x_1 - x_2}\vert<M \frac{\varepsilon}{M} = \varepsilon\) 由于柯西判别法,存在极限且极限为 \(1\),则极限存在且存在唯一定义 \(a ^ {x_0} = \lim\limits_{x\rightarrow x_0, x \in \mathbb Q} a ^ x\)

定理(指数函数的性质)

\(a > 1\)

  1. \(a ^ x\) 严格单调递增
  2. \(a ^ {x_1 + x_2} = a^{x_1} a ^ {x_2}\)
  3. \(\lim\limits_{x\rightarrow x_0} a ^ x = a ^ {x_0}\)
  4. \(\forall y > 0, \exists x \in \mathbb R, y = a ^ x\)

指数函数有时候记称 \(\exp_a(x)\),当 \(a = e\) 时候,直接写成 \(\exp(x)\)

对数函数和幂函数

定义: \(a>0,a\ne 1,\exp_a\R\rightarrow\R^+\) 的逆函数称为对数函数 \(\log_ax,x\in\R^+\)\(a=e\) 时记为 \(\ln x\)\(\log x\)

定理:

  1. \(a^{\log_ax}=x,\log_aa^x=x,\log_a1=0\)
  2. \(\log_ax_1x_2=\log_ax_1+\log_ax_2\)
  3. \(a>1\) 时严格单调递增, \(0<a<1\) 时严格单调递减
  4. \(x_0>0,\lim\limits_{x\rightarrow x_0}\log_ax=\log_ax_0\)
  5. \(a,b>0,a\ne 1,\log_ab^x=x\log_ab\)
  6. \((a^x)^y=a^{xy}\)

由2可知, \(\lim\limits_{x\rightarrow x_0}\log_ax=\log_ax_0=\log_a\lim\limits_{x\rightarrow x_0}x\) ,称为连续性(初等函数即具有连续性的函数)

定义(幂函数): \(x^{\alpha}=e^{\alpha\ln x}\)

极限过程的其它形式

定义: \(X\sub\R\) 无上界, \(f\) 为定义在 \(X\) 上的函数, \(A\in\R\) ,若 \(\forall\varepsilon>0,\exists M>0,\text{s.t.} x\in X\cup[M,+\infty) ,|f(x)-A|<\varepsilon\) ,称 \(x\rightarrow\infty\)\(f\) 的极限为 \(A\) ,记 \(\lim\limits_{x\rightarrow+\infty}f(x)=A\)

例2

\(a > 1\) 证明:\(\lim\limits_{x\rightarrow+\infty} a ^ {-x} = 0\)

\(a = 1 + \alpha > 1\)\(\frac{1}{a ^ x} \le \frac{1}{a ^ {[x]}} = \frac{1}{(1 + \alpha) ^ {[x]}}\le \frac{1}{\alpha[x] + 1}\rightarrow 0\)

例3

\(a > 0\)\(a \not = 1\)\(\lim \limits_{x \rightarrow + \infty} \frac{\log _{a} x}{x} = 0\)

证明:

  1. \(a > 1\) 的时候,令 \(y = \log_a x, \lim\limits_{x\rightarrow +\infty}\frac{\log_a x}{x} = \lim\limits_{y\rightarrow +\infty} \frac{y}{a ^ y} =\lim\limits_{y\rightarrow +\infty} \frac{y}{(1 + \alpha)^ y} \le \lim\limits_{y\rightarrow +\infty}\frac{y}{1 + \alpha[y] + \frac{[y][y - 1]}{2} \alpha ^ 2} \rightarrow 0\)
  2. \(0 < a < 1\) 时,\(\lim\limits_{x\rightarrow +\infty} \frac{\log _a x}{x} = \lim\limits_{x\rightarrow +\infty} -\frac{\log_{\frac{1}{a}}{x}}{x} = -\lim\limits_{x\rightarrow +\infty} \frac{\log_{\frac{1}{a}}{x}}{x}=0\)

无穷大量和无穷小量

定义

\(X\subset \mathbb R\)\(f\) 为定义在 \(X\) 上的函数,\(x_0\)\(X\) 的极限点。

定义:\(\forall A > 0,\exists \delta > 0, \text{s.t.}\forall x\in \mathring{U}_X (x_0, \delta)\),均有 \(f(x)>A\),称 \(x\rightarrow x_0\) 时,\(f(x)\) 趋近于 \(+\infty\),记作 \(\lim\limits_{x\rightarrow x_0} f(x) = + \infty\)

\(X \subset \mathbb R\) 无上界(下界) \(\lim\limits_{x\rightarrow +\infty} f(x) = +\infty\)

无穷小量和无穷大量

  1. \(\lim\limits_{x\rightarrow x_0} f(x) = 0\),那么 \(f(x)\)\(x\rightarrow x_0\) 为无穷小量
  2. \(\lim\limits_{x\rightarrow x_0} f(x) = \infty\),那么 \(f(x)\)\(x\rightarrow x_0\) 为无穷大量

相比较无穷小

定义 \(f, g\) 满足定义,\(\exists \delta > 0\), \(\exists\) 定义在 \(\mathring{U}_X(x_0, \delta)\) 上的函数 \(\alpha(x)\) 满足

  1. \(f(x) = \alpha(x) g(x),\forall x \in \mathring{U}_X (x_0, \delta)\)
  2. \(\lim\limits_{x\rightarrow x_0} \alpha(x) = 0\)

\(f(x)\) 相对于 \(g(x)\) 是无穷小量。记为 \(f(x) = o(g(x))\)

更高阶的量

定义 \(f, g\) 满足定义,\(\exists \delta > 0\), \(\exists\) 定义在 \(\mathring{U}_X(x_0, \delta)\) 上的函数 \(\alpha(x)\) 满足:

  1. \(f(x) = o(g(x))\)
  2. \(\lim\limits_{x\rightarrow x_0}g(x)=0\)

称为 \(f(x)\)\(x \rightarrow x_0\) 时候比 \(g(x)\) 是高阶无穷小量。

定义 \(f, g\) 满足定义,\(\exists \delta > 0\), \(\exists\) 定义在 \(\mathring{U}_X(x_0, \delta)\) 上的函数 \(\alpha(x)\) 满足:

  1. \(f(x) = o(g(x))\)
  2. \(\lim\limits_{x\rightarrow x_0}g(x)=+\infty,\lim\limits_{x\rightarrow x_0}f(x)=+\infty\)

称为 \(g(x)\)\(x \rightarrow x_0\) 时候比 \(f(x)\) 是高阶无穷大量。

比如说 \(a ^ x\)\(x ^ a\) 高阶。

同阶无穷量

定义 \(f, g\) 满足定义,\(\exists \delta > 0\), \(\exists\) 定义在 \(\mathring{U}_X(x_0, \delta)\) 上的函数 \(\alpha(x)\) 满足:

  1. \(\beta(x)\) 有界
  2. \(f(x) = \beta(x) g(x)\)

记为 \(f(x) = O(g(x))\),若同时有 \(g(x) = O(f(x))\),那么这两个是同阶的,特别的,如果和 \(\vert x - x_0 \vert ^ \alpha\) 同阶,那么称其为 \(\alpha\) 阶无穷小

渐进等价

定义 \(f, g\) 满足定义,\(\exists \delta > 0\), \(\exists\) 定义在 \(\mathring{U}_X(x_0, \delta)\) 上的函数 \(\alpha(x)\) 满足:

  1. \(\lim\limits_{x\rightarrow x_0} \alpha(x) = 1\)
  2. \(f(x) = \alpha(x)g(x)\)

\(f\)\(x\rightarrow x_0\) 的时候是渐进等价的,记为 \(f(x) \sim g(x), x\rightarrow x_0\)

这是一种等价关系

命题:\(f, g, h\)\(X\rightarrow R\) 的函数,\(x_0\)\(X\) 的极限,若

  1. \(f\sim g,(x\rightarrow x_0)\)
  2. \(\lim\limits_{x\rightarrow x_0} f(x) h(x)\) 或者 \(\lim\limits_{x\rightarrow x_0} g(x) h(x)\) 有一个存在

\(\lim\limits_{x\rightarrow x_0} f(x) h(x)=\lim\limits_{x\rightarrow x_0} g(x) h(x)\)

  1. \(o(f(x)) + o(f(x)) = o(f(x)), O(f(x)) + O(f(x)) = O(f(x))\)
  2. \(o(f(x)) + O(f(x)) = O(f(x)), o(f(x)) = O(f(x))\)
  3. \(g\cdot o(f(x)) = o(g\cdot f(x)), g\cdot O(f(x)) = O(g\cdot f(x))\)

以证明:

\(e ^ x = 1 + x + \frac{1}{2!} x ^ 2 + \ldots + \frac{1}{n!} +o(x ^n)\)

\(\log (1 + x) = x - \frac{1}{2}x ^ 2\)

连续函数

连续函数的定义

定义:\(f\) 定义在 \(X\subset \mathbb R\) 上的函数,\(x_0 \in X,\forall \varepsilon > 0, \exists \delta > 0, \text{s.t.} \forall x \in (x_0-\delta,x_0 + \delta)\),满足 \(f(x)\in (f(x_0) - \varepsilon, f(x_0) + \varepsilon\)

\(f\)\(x_0\) 点连续

\(f\)\(x_0\) 点连续也可以定义为: \(\forall \varepsilon > 0, \exists \delta > 0, \forall x \in X, \vert x - x_0 \vert < \delta\),满足 \(\vert f(x) - f(x_0) \vert < \delta\)

注:

  1. 如果 \(x_0\) 是极限点,\(f\)\(x_0\) 处连续,当且进当 \(\lim\limits_{x\rightarrow x_0} f(x) = f(x_0)\)
  2. 否则如果是孤立点,那么 \(\exists \delta > 0, \text{s.t.} U_X(x_0,\delta) =\{x_0\}\) 孤立点就是连续点

定义: \(f\) 有定义, \(x\subset \mathbb R\) 上的函数,成 \(f\)\(X\) 上连续,当且仅当在 \(X\) 上每一点连续, \(C(X) = \{f \vert f为X上的连续函数\}\)

连续函数的例子

  1. \(f(x) = c, \forall x \in \mathbb R\)
  2. \(f(x) = x, \forall x \in \mathbb R\)
  3. \(f(x) = \sin x,\forall x \in \mathbb R\)
  4. \(a > 0, a\not = 1\), \(a ^ x\) 是连续函数,\(\lim\limits_{x\rightarrow x_0} a ^ x = a ^ {x_0}\)
  5. \(f:\mathbb N \rightarrow \mathbb R, n\mapsto f(n) = a_n\) 为连续函数

间断点和其分类

\(f\) 在某个点上不连续,这个点就是间断点。

例一:\(D(x) = \begin{cases}1 & x \in \mathbb Q\\ 0 & x \in \mathbb R \backslash \mathbb Q\end{cases}\)

例二:\(sgn(x) = \begin{cases}1 & x > 0 \\ 0 & x = 0 \\ -1 & x < 0\end{cases}\)

定义:设 \(f\) 为定义在 \(X\) 上的函数,\(x_0\)\(f\) 的间断点

  1. \(x_0\) 是第一类间断点,若 \(f(x_0-),f(x_0+)\) 均存在
    1. \(x_0\) 为可去间断点,若 \(f(x_0-)=f(x_0+) \not = f(x)\)
    2. \(x_0\) 为跳跃间断点,若 \(f(x_0-)\not = f(x_0+)\),称 \(\vert f(x_0+) - f(x_0-)\)\(f\)\(x_0\) 上的跃度(jump)
  2. \(x_0\)\(f\) 的第二类间断点,若 \(f(x_0-),f(x_0+)\) 至少一个不存在

例子:

  1. \(f(x) = \frac{1}{x}, x_0 = 0\)\(f\) 的第二类间断点
  2. \(f(x) = \sin \frac{1}{x}\)
  3. \(f(x) = \begin{cases} \frac{\sin x}{x} & x \not = 0\\ 2 & x \not = 0\end{cases}\),此时 \(x = 0\) 为第一类间断点

注:\(f:X\rightarrow \mathbb R, g:X\rightarrow \mathbb R\)

  1. \(f(x) = g(x), x \not = x_0\)
  2. \(\lim\limits_{x\rightarrow x_0} g(x) = g(x_0)\)

\(x_0\)\(f(x)\) 的可去间断点

单调函数的间断点的性质

命题:设 \(f\) 为定义在 \(X\) 上的单调函数,那么

  1. \(f\) 的间断点只能是跳跃间断点
  2. \(f\) 的间断点是之多可数个

这是比较好证明的

连续函数的局部性质

命题:如果 \(f\) 是定义在 \(X\) 上的函数,在 \(x_0\) 连续

  1. \(\exists \delta > 0, f\)\(U_(x_0, \delta)\) 上有界
  2. \(f(x_0)>0\),则 \(\exists \delta > 0, \text{s.t.}f(x) > 0, x \in U_X(x_0, \delta)\)

定理:\(f, g:X\rightarrow \mathbb R\)\(f, g\)\(x_0\) 上连续,

  1. \(\forall \alpha, \beta \in \mathbb R, \alpha f + \beta g\)\(x_0\) 上连续
  2. \(f\cdot g\)\(x_0\) 上连续
  3. \(\exists\delta > 0, \forall x \in U_X(x_0, \delta), g(x) \not = 0, \frac{f}{g}\)\(x_0\) 上连续

命题 \(f:X\subset R\rightarrow Y \subset \mathbb R\)\(x_0\) 连续,\(g:Y\subset R\rightarrow Z \subset \mathbb R\)\(x_0\) 连续,那么 \(g\circ f\)

左右极限函数

定义:\(f\) 为定义在 \(X\) 上的函数

  1. \(f\) 限制在 \(X \cap (-\infty, x_0)\)\(x_0\) 点连续,那么称 \(f\)\(x_0\) 点左连续,右连续同理。
  2. 介值定理,设 \(f\in C([a, b])\)
    1. 若 $f(a)f(b) < 0,则 \(\exists c \in [a, b], \text{s.t.} f(c) = 0\)
    2. \(f(a)\not = f(b), \forall y\)\(f(a)\)\(f(b)\) 之间,存在 \(x_0\),\(\text{s.t.}f(x_0) = y\)

一些例子

  1. \(f:[0,1]\rightarrow [0, 1]\),\(\exists x, f(x) = x\)
  2. 奇数次多项式一定有实根

单调函数的一些性质

推论:\(f\in C([a, b])\)\(f\) 是单射 \(\Leftrightarrow f\) 为严格单调递增函数

证明:

\(\Leftarrow\):显然

\(\Rightarrow\):反证法

命题:\(f\in C([a, b])\),且 \(f\) 单调,则 \(f([a,b]) = \begin{cases} \{f(a)\} &f(a) = f(b) \\ [f(a), f(b)] & f(a) < f(b) \\ [f(b), f(a)] & f(b) < f(a)\end{cases}\)

Weierstrass 最值定理

定理:若 \(f\in C([a, b])\) 则一定存在 \(x_1, x_2 \in [a, b],\text{s.t.}, f(x_1) = \sup\limits_{x\in [a, b]} f(x), f(x_2) = \inf\limits_{x\in [a, b]} f(x)(f(x_1) = \max\limits_{x\in [a, b]} f(x), f(x_2) = \min\limits_{x\in [a, b]} f(x))\)

证明:

先证明有界性:

\(\forall \varepsilon > 0, \exists \delta_x > 0, \forall l \in U_{[a, b]}(x,\delta_x)\),满\(\vert f(x) - f(l)\vert < \varepsilon\),由于有限覆盖定理,那么 \(\vert f(x_1)- f(x_2)\vert < N \varepsilon\)

第二步是反证法,如果 \(M = \sup\limits_{x\in [a, b]}f(x), \forall x \in [a, b], M > f(x)\),则令 \(F(x) = \frac{1}{M - f(x)}\),且有 \(F \in C([a, b])\),从而 \(F(x)\)\([a, b]\) 上有上界 \(\mu\),那么 \(f(x) \le M - \frac{1}{\mu}\),这和 \(M = \sup\limits_{x\in [a, b]} f(x)\) 矛盾,于是存在 \(x_1, f(x_1) =\sup\limits_{x\in [a, b]} f(x)\),最小值情况可以类似证明

注:\(\mathbb R\) 上有界闭集成为紧集,紧集上的连续函数存在最值定理

一致连续性

\(f\)\(X\) 上连续,\(x_0 \in X, \forall \varepsilon > 0, \exists \delta = \delta(x_0, \varepsilon) > 0, \text{s.t.} \forall U_{X} (x_0, \delta), \text{s.t.} \vert f(x) - f(x_0) \vert < \varepsilon\)

如果在 \(X\) 上一致连续,那么 \(\forall x_0 \in X,\forall \varepsilon > 0, \exists \delta = \delta(\varepsilon) > 0, \text{s.t.} \forall x \in U_{x}(x_0, \delta),\text{s.t.} \vert f(x) - f(x_0) \vert < \varepsilon\)

\(\exists \varepsilon_0 > 0, \forall n, \exists s_n, t_n \in X, \vert s_n - t_n\vert \ge \varepsilon_0\)

定理(Heine-Cantor)

\(f(x)\in C([a, b])\),则 \(f\)\([a, b]\) 上一致连续

证明:先取有限覆盖,邻域大小是 \(\frac{1}{2}\delta\),里面都是然后 \(\delta =\frac{1}{2}\min\{\delta_1, \delta_2, \ldots, \delta_n\}\),然后对于 \(\forall x_0 \in X, x\in U(x_k, \frac{1}{2} \delta_{x_k})\),然后 \(\forall \vert x_1 - x_0 \vert < \delta\),那么在一个邻域里面,则 \(\vert f(x_1) - f(x_0)\vert < \varepsilon\),证毕。

例题:设 \(f\in C[a, +\infty)\)\(\lim\limits_{x \rightarrow +\infty}f(x) = A \in \mathbb R\),那么 \(f\)\([a, +\infty)\) 上一致连续

振幅

\(f:X\rightarrow \mathbb R,x_0\)\(X\) 的极限点,\(\delta > 0\),令 \(\omega(f;x_0, \delta) = \sup\limits_{x \in U_X(x_0, \delta)}f(x) - \inf\limits_{x \in U_X(x_0, \delta)}f(x) = \sup\limits_{x, y \in U_X(x_0, \delta)}\vert f(x) - f(y) \vert\)\(f\)\(U_X(x_0, \delta)\) 上的振幅。

\(\omega (f; x_0) = \lim\limits_{\delta \rightarrow 0+} \omega(f; x_0, \delta)\) 称其为 \(f\)\(x\rightarrow x_0\) 时的振幅。

命题:\(f\)\(x_0\) 连续的条件是 \(\omega(f; x_0) = 0\),反例:\(\sin\frac{1}{x}\)

\(M(x) = \sup\limits_{x\in [a, x]} f(x), m(x) = \inf\limits_{x\in [a, x]} f(x)\),都是连续函数

函数的上下极限

定义:\(f:X\rightarrow \mathbb R,x_0\)\(X\) 的极限点

\(E = \{a\in \mathbb R_{\infty}\vert \exists\{x_n\} \in X\backslash\{x_0\},\lim\limits_{n\rightarrow} x_n = x_0,\lim\limits_{x\rightarrow x_0} =(x) = a\}\)

\(\sup E, \inf E\) 分别是函数在 \(x_0\) 的上下极限,记为 \(\lim\limits_{x\rightarrow x_0} \sup f(x) = \sup E,\lim\limits_{x\rightarrow x_0} \inf f(x) = \inf E\)

定理:

  1. \(\sup E, \inf E\in E\)
  2. \(y > \sup E\),则 \(\exists\delta > 0, \text{s.t.} _\forall x \in \mathring{U}_X (x_0, \delta), f(x) < y\)
  3. \(y <\sup E\),则 \(\forall \delta > 0, \text{s.t.} \exists x_n \in \mathring{U}_X (x_0, \delta), f(x_n) < y\)
  4. \(y < \inf E\),则 \(\exists\delta > 0, \text{s.t.} \forall x\in \mathring{U}_X (x_0, \delta), f(x) < y\)
  5. \(y > \inf E\),则 \(\forall\delta > 0, \text{s.t.} \exists x_n \in \mathring{U}_X (x_0, \delta), f(x_n) < y\)

命题:

  1. \(\lim\limits_{x\rightarrow x_0} \sup f(x) = \lim\limits_{\delta \rightarrow 0 ^ +, x \in U_X (x_0, \delta)} \sup f(x)\)
  2. \(\lim\limits_{x\rightarrow x_0} \inf f(x) = \lim\limits_{\delta \rightarrow 0 ^ +, x \in U_X (x_0, \delta)} \inf f(x)\)
  3. \(\lim\limits_{x\rightarrow x_0} \sup f(x) - \lim\limits_{x\rightarrow x_0} \inf f(x) =\mathring{\omega}(f, x_0)\)

第三章 导数

导数的定义

定义:

  1. \(f :X\rightarrow \mathbb R\) 上的函数,\(x_0\)\(X\) 的极限点,若 \(\exists A(x_0)\),则 \(f(x + l) - f(x_0) = A(x_0)h+\alpha (x, h) (*), h\rightarrow 0\),其中 \(x + l \in X, \alpha(x, h) = o(h)\),我们称 \(f\)\(l\) 处可微的
  2. \(\delta x = h, \delta f(x) = f(x_0 + h) - f(x_0)\) 为自变量和增量,称线性映射 \(\mathbb R \rightarrow \mathbb R, h\mapsto A(x_0)h\)\(x_0\) 处的微分,记作 \(\mathrm{d} f(x0)\),从而 \(\mathrm{d} f(x_0)(h) = A(x_0)h\),则 \((*)\) 可以表示为 \(f(x_0 + h) - f(x_0) = \mathrm d f(x_0) (h) + \alpha(x_0, h),h\)
  3. \((*), A(x_0) = \lim\limits_{h\rightarrow 0}\frac{f(x_0 + h) - f(x_0)}{h}\) ,称 \(A(x_0)\)\(f\)\(x_0\) 处的倒数,记为 \(f'(x_0)\)
  4. 特别的 \(f(x) = x\) 时,\(f'(x) = 1, \mathrm d x: \mathbb R\y_nrightarrow \mathbb R, h\mapsto \mathrm d x(h) = h\),对于一般的函数 \(f\),有 \(f'(x) = \frac{\mathrm{ d f(x)(h)}}{\mathrm d x(h)} = \frac{\mathrm d f(x)}{\mathrm d x}\),称之为微商,函数 \(f\)\(x_0\) 点处的微分 \(d f(x_0) = f'(x_0) \mathrm d x\)
  5. \(f\)\(x_0\) 处可微,称 \(y = f(x_0) + f'(x_0) (x-x_0)\),为函数 \(f\) 在图像 \((x_0,f(x_0)\) 的切线,斜率是 \(f'(x_0)\)

注:\(\mathrm d f(x):\mathbb R \rightarrow \mathbb R\),但是实际上

  1. \(f:X \rightarrow \mathbb R\)\(f\)\(X\) 的每一点都可微分那么称 \(f\)\(X\) 上可微,记作 \(C ^ 1(X) = \{f\vert f 在 X 上可微\}\)

左右导数可以类比左右极限定义

例一:

\(f:X\rightarrow R\)\(0\) 处可微,且 \(f(0) = 0\)\(y_n = \sum_{k = 1} ^ n \sum_{k = 1} ^ n f(\frac{k}{n ^ 2})\)

\(\lim\limits_{n\rightarrow \infty} y_n = \frac{1}{2} f'(0)\)

证明:

\(f(x) = f(0) + f'(0)x + \beta(x)x\),且在 \(x \rightarrow 0, \beta(x) \rightarrow 0\)

\(y_n = \frac{1}{2}\left(1 + \frac{1}{n}\right) f'(0) + \sum\limits_{k = 1} ^ n \beta(\frac{k}{n ^ 2}) \frac{k}{n ^ 2}\)

\(n\rightarrow \infty\) 即可。

导数四则运算

\(f, g\)\(x_0\) 处可导的函数

  1. \((\alpha f + \beta g)' (x_0) = \alpha f'(x_0) + \beta g'(x_0)\)
  2. \((f\cdot g) '(x_0) = f'(x_0)g(x_0) + f(x_0) g'x(x_0)\)
  3. \(\left(\frac{f}{g}\right)'(x_0) = \frac{f'(x_0)g(x_0) - g'(x_0) f(x_0)}{g ^ 2`(x_0)}\)

复合函数的法则

定理:设 \(f:X\rightarrow Y, g:Y\rightarrow \mathbb R\) 并且 \(f\)\(x_0\) 可导,\(g\)\(f(x_0)\) 可以导,那么 \((g\circ f) '(x_0) = g'(f(x_0)) f'(x_0)\)

例:双曲函数

\(\sinh (x) = \frac{e^x - e ^ {-x}}{2},\cosh x = \frac{e ^ x + e ^ {-x}}{2},\tanh x = \frac{\sinh x}{\cosh x}\)

\((\sinh x)' = \cosh x, (\cosh x)' = \sinh x, \tanh x = \frac{1}{\cosh ^ 2 x},\cosh ^ 2x - \sinh ^ 2 x = 1,(\coth x)' = -\frac{1}{\sinh ^ 2 x}\)

逆函数

\(X, Y\subset \mathbb R, f:X\rightarrow Y\),在 \(x_0\) 点可微,且 \(f'(x_0) \not = 0\),那么其逆函数 \(f^{-1}\) 存在且在 \(y = f(x_0)\) 处是连续,则 \(f^{-1}\)\(y = f(x_0)\) 处可微且 \(f ^ {-1}(y_0) = \frac{1}{f'(x_0)}\)

逆函数对应求导的例子

\(\arcsin'(x) = \frac{1}{\cos y} = \frac{1}{\sqrt{1 - \sin ^ 2 y }}=\frac{1}{\sqrt{1 - x ^ 2}}\)

高阶导数

\(f:X\rightarrow \mathbb R\) 上可微,若 \(f':X\rightarrow \mathbb{ R}\) 仍然可微,称 \(f\) 二阶可微,记作 \(f''(x)\) 或者是 \(\frac{\mathrm{d}^2 f(x)}{d x ^ 2}\) ,如果是 \(n\) 阶可微,记成 \(f ^ {(n)}(x)\) 或者是 \(\frac{\mathrm{d}^n f(x)}{\mathrm{d} x ^ n}\),定义,如果 \(f\)\(X\)\(n\) 阶可微,且 \(f ^ {(n)} (x)\)\(X\) 上连续,那么称 \(f\)\(X\)\(n\) 阶连续可微,记作 \(C ^ {n} (X)\),若无穷阶可微,那么记作 \(C^{\infty}(X)\),例如 \(e ^ x, \sin x\)

定理(leibniz): 设 \(f, g \in C^ n(X)\),则 \(*(f\cdot g) ^ {(n)} (x) = \sum\limits_{k = 0} ^ n \binom{n}{k} f ^ {(k)}(x) g ^ {(n - k)}(x)\)

证明采取数学归纳法,是显然的

微分中值定理

函数的极值

定义:设 \(\delta > 0, f: U (x_0, \delta) \rightarrow \mathbb R\)

  1. \(\forall x \in \mathring{U}(x_0, \delta), f(x) \le (f_{x_0})\),那么 \(x_0\) 是极大值点,极小值点是类似的
  2. \(\forall x \in \mathring{U}(x_0, \delta), f(x) < (f_{x_0})\),那么 \(x_0\) 是严格极大值点,严格极小值点是类似的

定理(Fermat, 1688)

\(f\)\((a, b)\) 上可微,那么 \(x_0 \in (a, b)\) 为极值点,则 \(f'(x_0) = 0\)

证明:就是找个 \(h\) 左极限是大于等于 \(0\),右极限是小于等于 \(0\),于是极限等于 \(0\),则导数是 \(0\)

定义:导数为 \(0\) 的点是驻点

  1. 可疑极值点:驻点和不可导点

中值定理(罗尔拉格朗日柯西皮亚诺)

  1. 罗尔中值定理,设 \(f \in C([a, b]) \cap C^{1}((a, b))\)\(f(a) = f(b)\),则存在 \(\xi\in (a, b),\text{s.t.}f'(\xi) = 0\)
  2. 拉格朗日中值定理,设 \(f \in C([a, b]) \cap C^{1}((a, b))\),则存在 \(\xi \in (a, b),\text{s.t.} f'(\xi) = \frac{f(b) - f(a)}{b - a}\)

推论:

  • \(f \in C ^ 1(a, b)\)\(f'(x)\ge 0\),则 \(f\) 单调递增,小于等于 \(0\) 是同理的
  • \(f \in C ^ 1(a, b), f'(x) = 0\),则 \(f(x) = C\)
  • \(f, g \in C ^ {1}(a, b)\) 满足 \(f'(x) = g'(x),\forall x \in (a, b),f(x) = g(x) + C\)

\(f\in C ^ 1(a, b)\)\(\vert f'(x) \vert M\)\(f\)\((a, b)\) 上一致连续。

  1. 柯西中值定理:\(f, g \in C([a, b])\cap C^1((a, b))\)\(\exists \xi \in (a, b), \text{s.t.} f'(\xi)(g(b) - g(a)) = g'(\xi)(f(b) - f(a))\) 并且 若 \(g'(x) \not = 0,\forall x \in (a, b)\)\(\frac{f'(\xi)}{g'(\xi)} = \frac{f(b) - f(a)}{g(b) - g(a)}\)

\[F(x) =\begin{vmatrix} f(x) & g(x) & 1 \\ f(a) & g(a) & 1 \\ f(b) & g(b) & 1 \end{vmatrix} \]

  1. 皮亚诺中值定理:

\[f, g,h\in C([a, b]) \cap C^1((a, b)), \exists \xi \in (a, b), \text{s.t.} \begin{vmatrix} f'(\xi)&g'(\xi)&h'(\xi) \\ f(a) & g(a) & h(a)\\ f(b) & g(b) & h(b) \end{vmatrix} = 0 \]

证明:

\[F(x) = \begin{vmatrix} f'(\xi)&g'(\xi)&h'(\xi) \\ f(a) & g(a) & h(a)\\ f(b) & g(b) & h(b) \end{vmatrix} \]

  1. \(h(x) = 1\) 是柯西中值定理
  2. \(h \equiv 1, g(x) \equiv 1\) 是拉格朗日中值定理

例一

\(f\in C([a, b]) \cap C^2((a, b))\)\(f(a) = f(b) = 0\)\(\forall x \in (a, b),\exists \xi \in (a, b),\text{s.t.} f(x) = \frac{1}{2}f''(\xi) (x - a)(x - b)\)

例二

\(\delta > 0, f: U(a, \delta)\rightarrow \mathbb R\),可微,在 \(a\) 点二次可微,且 \(f''(a) \not = 0\),当 \(h\rightarrow 0\)\(f(a + h) - f(a) = f'(a +\theta (h) h)h\),其中 \(\lim\limits_{h\rightarrow 0}\theta (h) = \frac{1}{2}\)

证明:由 Lagrange 中值订立可以得到 \(f(a + h) - f(a) = f'(a + \theta(h)h)h,0<\theta(h) < 1\)

下面证明 \(\lim\limits_{h\rightarrow 0}\theta (h) = \frac{1}{2}\)

(Darbowx)定理:\(f \in C^1((a, b)), f'(x)\)\((a, b)\) 上具有介值定理

用导数研究函数的性质

定理:设 \(f\)\((a, b)\) 上可微,在 \((a, b)\) 单调递增的充要条件是 \(\forall x \in (a, b), f'(x) \ge 0\),严格单调递增的充要条件是 \(\forall x \in (a, b) f'(x) \ge 0\),并且不存在一个子区间使得 \(f'(x) \equiv 0\)

定理:(必要条件). 设 \(f\) 为定义在 \(U(x_0, \delta)\) 上的函数, 其中 \(\delta\) > 0. 则 \(x_0\)\(f\) 的极值点的必要条件是: 要么 \(f\)\(x_0\) 处不可微, 要么 \(f\)\(x_0\) 处可微且 \(f′(x_0) = 0\)

定理:设 \(f\)\(\mathring{U}(x_0, \delta)\) 可微,且在 \(x_0\) 上连续,那么我们有如下结论:

  • 如果当 \(x\in (x_0 - \delta, x_0)\) 时,\(f'(x) > 0\)\(x \in (x_0, x_0 + \delta)\) 时,\(f'(x) < 0\),那么 \(x_0\)\(f\) 的一个严格极大(反之是极小)值点。
  • 如果当 \(x\in \mathring{U}(x_0, \delta)\) 时,\(f'(x) > 0\) 或者 \(f'(x) < 0\),那么不是极值点

定理:设函数 \(f:U(x_0, \delta)\rightarrow R\)\(x_0\) 处有直到 \(n\) 阶的导数,并且 \(f^{(1)}(x) = \cdots = f^{(n-1)}(x) =0,f^{(n)}(x) \ne 0\)

  • \(n\) 为奇数时 \(x_0\) 不是 \(f\) 的极值点
  • \(n\) 为偶数时 如果 \(f^{(n)}(x0) > 0\), 则 \(f\)\(x_0\) 处取到严格极小值; 而如果 \(f^{(n)}(x_0) < 0\), 则 \(f\)\(x_0\) 处取到严格极大值.

Young 不等式

\(a, b > 0, p, q \not = 0, 1, \frac{1}{p} + \frac{1}{q} = 1\)

  1. \(p > 1 , a^{\frac{1}{p}} b^ \frac{1}{q} \le \frac{a}{p} + \frac{b}{q}\)
  2. \(p < 1, a^{\frac{1}{p}} b^ \frac{1}{q} \ge \frac{a}{p} + \frac{b}{q}\)

并且 \(a = b\) 取到等式

\(H\ddot older\) 不等式

\(a_i \ge 0, b_i \ge 0, \frac{1}{p} + \frac{1}{q}= 1, p, q > 0\)

  1. \(p > 1, \sum a_i b_i \le \left(\sum a_i^ p\right) ^ {\frac{1}{p}}\left(\sum b_i^ q\right) ^ {\frac{1}{q}}\)
  2. \(p <1 , \sum a_i b_i \ge \left(\sum a_i^ p\right) ^ {\frac{1}{p}}\left(\sum b_i^ q\right) ^ {\frac{1}{q}}\)

Minkowski 不等式

\(a_i, b_i \ge 0\)

  1. \(p > 1, (\sum (a_i + b_i) ^ p)^{\frac{1}{p}} \le (\sum a_i ^ p) ^ {\frac{1}{p}}+(\sum b_i ^ p) ^ {\frac{1}{p}}\)
  2. \(p <1, (\sum (a_i + b_i) ^ p)^{\frac{1}{p}} \ge (\sum a_i ^ p) ^ {\frac{1}{p}}+(\sum b_i ^ p) ^ {\frac{1}{p}}\)

凸函数

凸函数的定义

\(f:X\subset R\rightarrow R\)

\(\forall x_1, x_2 \in X, \lambda \in (0, 1)\) 满足 \(f(\lambda x_1 + (1 - \lambda) x_2) \le \lambda f(x_1) + (1 - \lambda) f(x_2)\) 称之为凸函数,类似的定义严格凸函数。

等价定义:\(x < y < z, f\) 凸函数, \(\Leftrightarrow \det{\begin{bmatrix} 1 & 1 & 1 \\ x & y & z \\ f(x) & f(y) & f(z)\end{bmatrix}}\ge 0\)

\(-f\) 是凸函数,那么 \(f\) 就是凹函数

凸函数的性质

定理 设 \(f:(a, b) \rightarrow \mathbb R\) 为凸函数,则 \(f\in C((a, b))\)

可微凸函数的性质

定理:设 \(f:(a, b)\rightarrow \mathbb R\) 可微,则 \(f\) 是凸函数 \(\Leftrightarrow f'(x) 是增函数\)

若为严格凸函数 \(\Leftrightarrow f'(x) 在 (a, b) 上严格增函数\)

推论:若 \(f:(a, b) \rightarrow \mathbb R\)\(2\) 阶可微,

  1. \(f\)\((a, b)\) 上凸函数 \(\Leftrightarrow f''(x) \ge 0, x\in (a, b)\)
  2. 为严格凸函数 \(\Leftrightarrow f''(x) \ge 0,f''(x) \ge 0\), 且任何子区间上 \(f''(x)\) 不恒为 \(0\)

定理:设 \(f:(a, b) \rightarrow \mathbb R\) 可微,则

  1. \(f\)\((a, b)\) 上是凸函数 \(\Leftrightarrow\) 函数图像上任意一点均在切线上方
  2. \(f\)\((a, b)\) 上严格凸 \(\Leftrightarrow\) 函数图像上除了切点之外都在切线上方

注:若 \(f:(a, b) \rightarrow \mathbb R\) 为凸函数,则 \(\forall x_0 \in (a, b), \exists k(x_0),\text{s.t.} f(x) \ge f(x_0) + k(x_0)(x - x_0)\)

\(y = f(x_0) + k(x_0)(x - x_0)\)\(f\)\(x_0\) 上的支撑线 \((x_0, f(x_0))\) 称为支撑点

用凸函数凹函数考察不等式

例一 Jensen不等式

\(f: (a, b) \rightarrow \mathbb R\) 上的凸函数,满足 \(\lambda_i \ge 0, \sum \lambda_i =1\),则对于 \(\forall x_1, \ldots, x_n \in (a, b)\)

\(f(\sum \lambda_i x_i) \le \sum \lambda_i f(x_i)\)

  1. \(f:(a, b)\rightarrow \mathbb R\) 是凸函数,则 \(\lim\limits_{x\rightarrow a^+} f(x), \lim\limits_{x\rightarrow b^-} f(x)\) 存在,若 \(f:(a, b)\rightarrow \mathbb R\) 有界且是凸函数,则 \(f\)\((a, b)\) 上一致连续。
  2. \(f:(a, b)\rightarrow \mathbb R\) 是凸函数, \(f\) 的不可微点至多可数个。
  3. \(f:(a, b)\rightarrow \mathbb R\) 是凸函数,则 \(\forall x \in (a, b), f'_-(x)\) 左连续,\(f'_+(x)\) 右连续。

L'Hospital 法则

定理:

  1. \(f, g:(a, b) \rightarrow \mathbb R\) 为可微函数,\(a\) 可以为 \(-\infty\), \(b\) 可以是 \(+\infty\)
  2. \(g'(x) \not = 0, \lim\limits_{x\rightarrow a^+} \frac{f'(x)}{g'(x)} = A\)
  3. \(\lim\limits_{x\rightarrow a ^ +} f(x) = 0\)\(\lim\limits_{x\rightarrow a ^ +} g(x) = 0\) 或者 \(\lim\limits_{x\rightarrow a ^ +} g(x) = \infty\)

则 $\lim\limits_{\rightarrow a ^ +} = \frac{f(x)}{ g(x)} = A}

函数做图

\(f:(a, b) \rightarrow \mathbb R\), \(a = -\infty\)\(b = +\infty\)

\(x\rightarrow -\infty\)\(x\rightarrow +\infty\)\(f(x) = kx + o(1)\),则称其为 \(y = kx + b\)\(f\)\(x\rightarrow -\infty\)\(x\rightarrow +\infty\) 的渐进线。

\(f:(a, b) \rightarrow \mathbb R\),若 \(\lim\limits_{x\rightarrow a ^ +} = \infty\) 或者是 \(\lim\limits_{x\rightarrow b ^ -} f(x) = \infty\) 那么称 \(x = a\)\(x = b\) 是竖直渐进线。

Taylor 公式

一元微积分的顶峰

带 Peano 余项的多项式

定义与定理

\(P_n(x_0; x) = a_0 + a_1(x - x_0) + \ldots + a_n(x - x_0) ^ n\)

\(P^{(k)}(x_0; x) = k! a_k, k = 0, 1, \ldots, n\)

于是可以知道 \(a_k = \frac{P ^ {(k)} _n (x_0; x)}{k!}\),设 \(f\)\(x_0\) 处有 \(n\) 阶导数,定义 \(T_n(x_0; x) = \sum\limits_{k = 0} ^ n \frac{f ^ {(k)}_n(x_0)}{k!} (x - x_0) ^ k\) 称之为 \(f\)\(x_0\) 点的 Taylor 多项式, \(f(x) = T_n(x_0; x) + r_n(x_0; x)\),称 \(r(x_0; x)\) 为余项。

定理:设 \(f:[a, b]\) 上的函数,在 \(x_0\in [a, b]\) 具有 \(n\) 阶导数,当 \(x\rightarrow x_0\) 时,\(f(x) = T_n(x_0; x) + o((x - x_0) n)\)

\(x = 0\) 是 Maclawrin 展开。

带有 Lagrange 余项和 Cauchy 余项的 Taylor 展开式

定理: 设 \(f\)\([x_0, x]\)\(n\) 阶连续可微,且在 \((x_0, x)\) 上有 \(n + 1\) 阶导数,那么

\[f(x) = \sum\limits_{k = 0} ^ n \frac{f^{(k)}(x_0)}{k!} (x - x_0) ^ k+\frac{f ^ {(n + 1) (\xi)}}{(n + 1)!}(x - x_0) ^ {n + 1} \]

其中 \(\xi \in (x_0, x)\)

例一

\(f(x) = e ^ x, f(x) = \sum \limits_{k = 0} ^ n \frac{x ^ k}{k!} + \frac{e ^ {\theta x}}{(n + 1)!}x ^ {n + 1} = \sum\limits_{k = 0} ^ {\infty} \frac{x ^ k}{k!}\)

\(\sin x = \sum\limits_{k = 0} ^ \infty \frac{(-1) ^ k}{(2k + 1)!}x ^ {2k + 1}\)

\(\cos x = \sum\limits_{k = 0} ^ n \frac{(-1) ^ k}{(2k)!} x ^ {2k} + (-1) ^ {n + 1} \frac{\cos(\theta x)}{(2n + 2)!} x ^ {2n + 2} = \sum\limits_{k = 0} ^ \infty \frac{(-1) ^ k}{(2k)!}x ^ {2k}\)

Taylor 级数

定义:设 \(f\)\(x_0\) 点无穷次可微,那么 \(\sum\limits_{k = 0} \frac{f ^ {(k)} (x_0)}{k!}(x - x_0) ^ k\),一般而言函数的 Taylor 级数未必收敛到本身。

\(f ^ {(n + 1)} (x) = P_{n + 1} \left(\frac{1}{x}\right) e ^ {\frac{-1}{x ^ 2}}\)

\(\lim\limits_{x\rightarrow 0} \frac{f ^ {(n)} (x)- f ^ {(n)}(0)} x = \lim_{x\rightarrow 0} \frac{ P _n(\frac{1}{x}) e ^ {-\frac{1}{x ^ 2}}}{x} = 0\)

于是 \(f ^ {(k)}(0) = 0\)

\(\vert f''(x)\vert \le M, \forall x \in [a, b], \vert f(x) - f(a) - \frac{f(b) - f(a)}{b - a} \vert \le \frac{M}{8} (b - a) ^ 2\)

Taylor 的唯一性

定理:设 \(T_n(x_0; x)\)\(f\) 的泰勒多项式,\(P_n = a_0 + a_1(x - x_0) + \cdots + a_n(x - x_0) ^ n\) 为唯一多项式,且 \(T_n(x_0; x) \not = P_n (x_0, x)\)\(\exists \delta > 0, \text{s.t.} 0 < \vert x - x_0\vert < \delta\)\(\vert f(x) - T_n (x_0; x) \vert < \vert f(x) - P_n (x_0; x)\vert\)

求导的逆运算

原函数的概念

定义:\(I = (a, b), [a, b], (a, b], [a, b)\)

\(F, f:I \rightarrow \mathbb R, F(x)\) 可以微,\(\forall x \in I\)\(\mathrm{ d} F(x) = f(x) \mathrm{d} x\)\(F(x)\)\(f(x)\) 的一个原函数。

定理 :\(F_1, F_2\)\(f\)\(I\) 上的原函数,则 \(F_1(x) = F_2(x) + C\) ,那么 \(F_1' - F_2' = 0\)

\(F(x)\)\(f\)\(I\) 上的一个原函数,\(\{F(x) + C\vert c \in \mathbb R\}\),定义,球一个函数 \(f\)\(I\) 上的原函数的运算叫做不定积分,记作 \(\int f(x) \mathrm{d} x\)

不定积分的性质

  1. \(\forall \alpha, \beta \in \mathbb R, \int (\alpha f(x) + \beta g(x))\mathrm{d}x = \alpha\int (f(x))\mathrm{d}x + \beta\int (g(x))\mathrm{d}x\)
  2. \(f, g\) 均可微

\[\int (fg)' \mathrm{d}x = \int (f'g + fg') \mathrm{d}x\\ \int (fg)' \mathrm{d}x = \int d(fg)(x) = f(x)g(x) + C \int f'g \mathrm{d}x = \int (fg)'\mathrm{d}x - \int g'f\mathrm{d}x \]

  1. 换元公式

\(I\stackrel {\phi} {\longrightarrow}J\stackrel {f} {\longrightarrow} \mathbb R\)

其中 \(I, J\) 区间 \(\phi\) 可微,\(F\)\(f\) 的一个原函数

\(\int (f\circ \phi)(t) \phi'(t) \mathrm{d}t = \int (f\circ \phi)(l) \mathrm{d}\phi(l) \stackrel{x = \phi(t)}{=} \int f(x) \mathrm{d}x = F(phi(t)) + C\)

几个基本公式

\[\begin{aligned} &\int 0 \mathrm{d}x = C \\ &\int x ^ {\alpha} \mathrm{d}x = \begin{cases} \frac{1}{\alpha + 1} x ^ {\alpha + 1}\\ \ln \vert x \vert \end{cases} \\ &\int e ^ x \mathrm{d}x = e ^ x + C\\ &\int a ^ x \mathrm{d}x = \frac{a ^ x}{\ln a} + C\\ &\int \sin x \mathrm{d}x = = -\cos x + C\\ &\int \cos x \mathrm{d}x = \sin x + C\\ &\int \frac{\mathrm{d}x}{1 + x ^ 2} = \arctan x + C\\ &\int \frac{\mathrm{d}x}{a ^ 2 + x ^ 2} = \frac{1}{a} \arctan x + C\\ &\int \frac{\mathrm{d}x}{\cos ^ 2 x} = \tan x + C\\ &\int \frac{\mathrm{d}x}{\sqrt{1 - x ^ 2}} =\arcsin x + C &\int \frac{-\mathrm{d}x}{\sqrt{1 - x ^ 2}} =\arccos x + C \end{aligned} \]

注:

  1. 连续函数均有原函数
  2. 出等函数的导函数是初等函数,但是初等函数的不定积分不一定是初等函数

分部积分和换元形式

分部积分

\(f, g\)\(I\) 上可微

\(\int f'(x)g(x) \mathrm{d}x = f(x) g(x) - \int f(x) g'(x) \mathrm{d}x\) 称为不定积分的分部积分

或者等价的有 \(\int f(x) \mathrm{d} g(x) = f(x) g(x) - \int g(x) \mathrm d f(x)\)

\[ \newcommand\m[2]{\int #1\mathrm{d} #2} \begin{aligned} &\int \ln x \mathrm{d}x = x\ln x - \int \frac{1}{x}x \mathrm{d}x = x\ln x - x \\ &\int x e ^ x \mathrm{d}x = x e ^ x - \int e ^ x \mathrm{d}x = x e ^ x - e ^ x\\ &\int e ^ x \sin (tx) \mathrm{d}x = \int \sin (tx) \mathrm{d e ^ x} = \sin (tx) e ^ x - t\int e ^ x \cos(tx) \mathrm{d}x = \sin (tx) e ^ x - t[e ^ x \cos (tx) + t \int e ^ x \sin (tx) \mathrm{ d}x ] \\ &\Rightarrow (1 + t ^ 2) \int e ^ x \sin (tx) \mathrm{d}x = e ^ x (\sin (tx) - t \cos (tx))\Rightarrow \int e ^ x \sin (tx) \mathrm{d}x = \frac{e ^ x}{1 + t ^ 2} (\sin (tx) - t \cos (tx)) \\ &\int \sin ^ n x = -\m{\sin ^ {n - 1}}(-\cos x) = -\sin ^{n - 1} x \cos x + \int \cos x \mathrm{d} \sin ^ {n - 1} x = -\sin ^{n - 1} x \cos x + (n - 1) \int \cos ^ x \sin ^ {n - 2} x \mathrm d x \\ &=-\sin ^ {n - 1} x\cos x + (n - 1) \int \sin ^ {n - 2} \mathrm d x - (n - 1) \int \sin ^ {n} x \mathrm d x \end{aligned} \]

于是我们得到

\(\int \sin ^ n x \mathrm{d}x = - \frac{1}{n}\sin ^ {n - 1} x\cos x+ \frac{n - 1}{n} \int \sin ^ {n - 2} x \mathrm{d} x\)

\(n = 1\) 时,\(\int \sin x \mathrm{d} x = -\cos x + C\)

\(n = 2\) 时,\(\int \sin ^ 2 x \mathrm{d} x =-\frac{1}{2} \sin x \cos x + \frac{1}{2} x + C\)

\(a > 2\) 考虑不定积分,

\[F = \int \frac{\mathrm{ d} x}{x ^ 4 + a x ^ 2 + 1}, G = \int\frac{x ^ 2 \mathrm{d}x}{x ^ 4 + a x ^ 2 + 1} \]

我们有

\[F + G = \int \frac{\left(1 + \frac{1}{x ^ 2}\right)\mathrm{d} x}{x ^ 2 + a + \frac{1}{x ^ 2}} = \int \frac{\mathrm{d} (x - \frac{1}{x})}{x ^ 2 + a + \frac{1}{x ^ 2}} = \frac{1}{\sqrt{a + 2}} \arctan \frac{x - \frac{1}{x}}{\sqrt{a +2}} + C \\ F - G = \int \frac{\left(1 - \frac{1}{x ^ 2}\right)\mathrm{d} x}{x ^ 2 + a + \frac{1}{x ^ 2}} = \int \frac{\mathrm{d} (x +\frac{1}{x})}{x ^ 2 + a + \frac{1}{x ^ 2}} = \frac{1}{\sqrt{a -2}} \arctan \frac{x +\frac{1}{x}}{\sqrt{a - 2}} + C \]

注:连续函数有原函数,反之不一定,比如说

\[F(x) = \begin{cases} x ^ 2 \sin \frac{1}{x}& x \not = 0\\ 0 & x = 0 \end{cases} ,f(x) = \begin{cases} 2x \sin \frac{1}{x} - \cos \frac{1}{x} & x\not = 0 0 & x = 0 \end{cases} \]

显然 \(x = 0\)\(f(x)\)\(x = 0\) 时,不连续。

分部函数函数的积分

因式分解

\[P(x) = a_n x ^ n+\ldots + a_0 \\ Q(x) = b_n x ^ m + \ldots + b_0\\ R(x) = \frac{P(x)}{ Q(x)} \]

  1. \(n \ge m, R(x)\) 称为假分式
  2. \(n < m, R(x)\) 称为真分式

\(n\) 次复系数多项式一定有根 \(\leftrightarrow\) \(n\) 复系数次多项式有 \(n\) 个复根。

若为实系数多项式:

\(Q(x) = q(x - x_1) ^ {r_1} (x - x_2) ^ {r_2} \ldots (x - x_k) ^ {r_k} (x ^ 2 + b_1x + c) ^ {s_1}\ldots (x^2 + b_l x + c_l) ^ {s_l}\)

\(q \in \mathbb R, \sum\limits_{j = 1} ^ k r + 2 \sum\limits_{j = 1} ^ l s = m, \delta_j = b_j ^ 2 - 4 c_j < 0\)

一般而言

\[Q(x) = Q_1(x)(x - a) ^ m \]

那么我们有

\[\frac{P(x)}{Q(x)} = \frac{P_1(x)}{Q_1(x)}\sum\limits_{k = 1} ^ m \frac{A_k}{(x - a) ^ k} \]

如果存在

\[Q(x) = Q_1(x)(x +bx + c) ^ m,b ^ 2 - 4c < 0 \]

那么我们有

\[\frac{P(x)}{Q(x)} = \frac{P_1(x)}{Q_1(x)}\sum\limits_{k = 1} ^ m \frac{A_k}{(x +bx+c) ^ k} \]

由因式分解,因该有

\[\begin{aligned} &\int \frac{\mathrm{d}x}{(x - a) ^ m} = \begin{cases} -\frac{1}{ m - 1}(x - a) ^ {-m + 1} + C & m \not = 1\\ \ln \vert x - a\vert + C & m = 1 \end{cases} \\ &\int \frac{2x \mathrm{d}x}{(x ^ 2 + a ^ 2) ^ m}= \int \frac{\mathrm{d} (x ^ 2 + a ^ 2)}{(x ^ 2 + a ^ 2) ^ m} = \begin{cases} -\frac{1}{ m - 1}(x ^ 2 + a ^ 2) ^ {-m + 1} + C & m \not = 1\\ \ln (x ^ 2 + a ^ 2)+ C & m = 1 \end{cases} \\ &\int \frac{\mathrm{d} x}{(x ^ 2 + a ^ 2) ^ m} = \frac{x}{(x ^ 2 + a ^ 2) ^ m} + 2m \int \frac{\mathrm{d}x}{(x^2+a^2)^{m+1}}\\ &=\frac{x}{(x ^ 2 + a ^ 2) ^ m} + 2m\int \frac{\mathrm{d} x}{(x^2+a^2)^m} - 2ma^2\int\frac{\mathrm{d}x}{(x ^ 2 +a ^ 2) ^ {m + 1}} \end{aligned} \]

于是我们得到递推式

\[\int \frac{\mathrm{d} x}{(x ^ 2 + a ^ 2) ^ {m + 1}} =\frac{1}{2ma ^ 2} \frac{x}{(x^2+a^2)^m}+\frac{2m - 1}{2ma ^ 2} \int\frac{\mathrm{d}x}{(x ^ 2 + a ^ 2) ^ m} \]

例子

\(R(x) = \frac{x ^ 2 + x - 1}{(x - 1) ^ 2 (x - 2)}\)

计算不定积分 \(\int R(x) \mathrm{d} x\)

解:

\(R(x) = \frac{A}{x - 1} + \frac{B}{(x - 1) ^ 2} + \frac{C}{x - 2}, C = \lim\limits_{x \rightarrow 2}(x - 2) R(x) = 7, B = \lim\limits_{x\rightarrow 1} (x - 1) ^ 2 R(x) = -3, A = \lim \limits_{x\rightarrow 1}(R(x) + \frac{3}{(x - 1) ^ 2}) = -6\)

\[\begin{aligned} &\int \frac{\mathrm{d}x }{x ^ 4 + 1} = \int \frac{\mathrm{d} x}{(x ^ 2 + \sqrt{2} x + 1)(x ^ 2 - \sqrt{2} x + 1)} =\int \left(\frac{x + \sqrt{2}}{2\sqrt{2}(x ^ 2 + \sqrt{2} x + 1)} - \frac{x - \sqrt{2}}{2 \sqrt{2} (x ^ 2 - \sqrt{ 2} x + 1)}\right)\mathrm{d} x \\ &= \int \left(\frac{x + \frac{\sqrt{2}}{2} + \frac{\sqrt{2}}{2}}{2\sqrt{2}(x ^ 2 + \sqrt{2} x + 1)} - \frac{x - \frac{\sqrt{2}}{2} - \frac{\sqrt{2}}{2}}{2\sqrt{2}(x ^ 2 -\sqrt{2} x + 1)}\right) \\ &=\frac{1}{4\sqrt{2}}\left(\log (x ^ 2 + \sqrt{2}x + 1) - \log (x ^ 2 - \sqrt{2} x + 1)\right) + \frac{\sqrt{2}}{4}\left(\arctan(\sqrt{2}x + 1) - \arctan(\sqrt{2}x - 1)\right) + C \end{aligned} \]

可化为有理函数

形如 \(\sum\limits_{i = 0}^n\sum\limits_{j = 0} ^m a_{ij} u ^ i v ^ j\) 称为二元多项式 \(R(u, v) = \frac{P(u, v)}{Q(u, v)}\)

使用万能公式,我们有 \(t = \tan \frac{x}{2}, \sin x = \frac{2t}{1 + t ^ 2}, \cos x = \frac{1 - t ^ 2}{1 + t ^ 2}, \mathrm{d} x = \frac{2\mathrm{d}t}{1 + t ^ 2}\)

\[\begin{aligned} &\int R(\cos x, \sin x) \mathrm{d}x \\ &\int R(\frac{1 - t ^ 2}{1 + t ^ 2}, \frac{2t}{1 + t ^ 2}) \frac{2 \mathrm{d} t} {1 + t ^ 2} \end{aligned} \]

\(n\) 中特殊情形:

  1. \(R(-u, v) = -R(u, v), \exists R_1, \text{s.t.} R(u, v) = R_1(u ^ 2 , v) u\),令 \(t = \sin x\), \(\int R(\cos x, \sin x) \mathrm{d}x = \int R_1 (\cos ^ 2 x, \sin x) \cos x \mathrm{d}x = \int R_1(1 - t ^ 2, t)\mathrm{d}t\)
  2. \(R(u, -v) = -R(u, v), \exists R_2, \text{s.t.} R(u, v) = R_2(u , v^2) v\),令 \(t = \cos x\), \(\int R(\cos x, \sin x) \mathrm{d}x = -\int R_2 (\cos x, \sin^2 x) \cos x \mathrm{d}x = \int R_1(1 - t ^ 2, t)\mathrm{d}t\)
  3. \(R(-u, -v) = R_3(u, v), \exists R_3, \text{s.t.} R(u, v) = R_3(u ^ 2, \frac{v}{u})\),令 \(t = cos x\)\(\int R(\cos x, \sin x ) \mathrm{d}x =-\int R_3(t, 1 - t ^ 2) {\mathrm{d}t}\)
    ]

例1

\[\int \frac{\sin ^ 5 x}{\cos ^ 4 x} \mathrm{d} x = -\int \frac{\sin ^ 4 x}{\cos ^ 4 x} \mathrm{d} \cos x\stackrel{y = \cos x}{=} -\int \frac{(1 - y ^ 2) ^ 2}{y ^ 4} \mathrm{d}y = \frac{1}{3 \cos ^ 3 x} + \frac{1}{\cos x} - \cos x + C \]

\[\int\frac{\mathrm{d}x}{\cos x + \sin x} = \int \frac{\mathrm{d}x}{\cos ^ 2 x(1 + \tan x) ^ 2} = \int \frac{\mathrm{d} \tan x}{1 + \tan ^ 2 x} \stackrel{y = \tan x}{=} \int \frac{\mathrm{d} y}{(1 + y)^2} = \frac{-1}{1 + y} + C = -\frac{\cos x}{\sin x + \cos x} + C \]

\[\int \frac{\mathrm{d}x}{2 + \cos x} \stackrel{t = \tan \frac{x}{2}}{=} \int \frac{1}{2 + \frac{1 - t ^ 2}{1 + t ^ 2}} \frac{2\mathrm{d}t}{1 + t ^ 2} = \int\frac{2\mathrm{d}t}{3 + t ^ 2} =\frac{2}{\sqrt{ 3}} \arctan \frac{t}{\sqrt{3}} + C \]

例2

\[\int R(x, \sqrt[m]{\frac{\alpha x + \beta}{\gamma x + \delta}}) \mathrm{d}x \]

其中 \(\alpha \delta - \beta\gamma \not = 0\)

\[\begin{aligned} t^m = \frac{\alpha x + \beta}{\gamma x + \delta} &\Leftrightarrow& x = \frac{\frac{\delta}{\alpha} t ^ m - \frac{\beta}{\alpha}}{ 1 - \frac{\gamma}{\alpha} t ^ m} = \frac{\delta t ^ m - \beta}{\alpha - \gamma t ^ m} = \phi(t)\\ \int R(x, \sqrt[m]{\frac{\alpha x + \beta}{\gamma x + \delta}})\mathrm{d}x = \int R(\phi(t), t) \phi'(t) \mathrm{d} t \end{aligned} \]

例3

\[\int \sqrt{\frac{x - 1}{x + 1}} \mathrm{d}x \stackrel{t ^ 2 = \frac{x - 1}{ x+ 1} \Leftrightarrow x = \frac{2}{1 - t ^ 2} - 1, \mathrm{d} x = \frac{4t \mathrm{d}t}{(1 - t ^ 2) ^ 2}}{=}\int \frac{4t ^ 2}{(1 - t ^ 2) ^ 2} \mathrm{d}t = \int \left(\frac{1}{t - 1} + \frac{1}{t + 1} + \frac{1}{(t - 1) ^ 2} + \frac{1}{(t + 1) ^ 2}\right) \mathrm{d} t = \ln |t - 1| - \ln |t + 1| - \frac{1}{t - 1} - \frac{1}{t + 1} + C \]

\[\int R(x, \sqrt{a x ^ 2 + bx + c}) \mathrm{d}x \\ ax ^ 2 + bx + c = a(x + \frac{b}{2a}) ^ 2 - \frac{b^2-4ac}{4a``} \]

此处中间有很多东西

椭圆积分

\(\int R(x, y) \mathrm{d}x\) \(y\)\(x\) 的代数函数。

\[P_0(y) x ^ n + \ldots + P_{n - 1}(x) x + P_n(y) = 0 \]

此处差点东西

一般而言这种东西可以微,那么有三类

\[\int \frac{\mathrm{d}x}{\sqrt{(1 - x ^ 2)(1 - k ^ 2 x ^ 2)}} \\ 此处应有两个 \]

函数的积分

积分的概念

引例

计算曲边梯形的面积

\(f\in C([a, b]), f(x) > 0, \forall x \in [a, b]\),计算 \(x = a, x = b, f(x), y = 0\) 围成的面积

  1. \(f = c, S = c(b - a)\)
  2. \(f = x, A = \frac{b ^ 2 - a ^ 2}{2}\)

希望对于一般的函数,也能做。

\(\xi_i \in [x_{i - 1}, x_i]\) \(\sums\limits_{i = 1} ^ n f(\xi_i)\delta_i\)

我们如果令 \(\lambda = \max(x_{i + 1} - x_i), \lim_{\lambda\rightarrow 0} f(\xi_i)(x_i - x{i - 1})\)

积分的定义

定义:\(f:[a, b]\rightarrow \mathbb R, I \in \mathbb R\)\(\varepsilon > 0, \exists \delta > 0, \text{s.t.}\) 对于 \([a, b]\) 的任意带标记点的分割 \((P, \xi), P:a = x_0 < x_1 < \ldots < x_{i} < \ldots < x_n = b\)

\(\Vert P = \Vert = \max(x_{i} - x{i - 1}) < \delta\) 都有 \(\vert \sum_{i = 1} ^ n f(\xi_i) (x_i - x_{i - 1}) - I \vert < \varepsilon\),称 \(f\)\([a, b]\) 上是 Rieman 可积分的,记为 \(\int_a ^ b f(x) \mathrm{d}x\)

注:\(\lim\limits_{\Vert P \Vert\rightarrow 0} \sum\limits_{i = 1} ^ n f(\xi _i)(x_i - x_{i - 1} ) = \int ^ b _a f(x) \mathrm{d}x,\delta x_i = x_i - x_{i - 1}, S (P, \xi) = \sum\limits_{i = 1} ^ n f(\xi_i) \delta x_i\) 称为带标记点的 Rieman 和,\([a, b]\) 称为积分区间

问题:

  1. 满足什么条件可以让函数可以积分

  2. 如何计算?

  3. \(f(x) = c, \int ^ b_a f(x) \mathrm{d}x = \int ^ b_ac \mathrm{d}x = c(b - a)\)

  4. \(f(x) = x, \int ^ b_a x\mathrm{d}x\),取 \(\xi_i = \frac{x_i + x_{i - 1}}{2}\),那么原来的式子是 \(=\frac{b ^ 2 - a ^ 2}{2}\),下面证明无论怎么取都对,\(\vert S(P, \eta) - \frac{b ^ 2 - a ^ 2}{2}\vert = |\sum_{i = 1} ^ n (\xi_i, \eta_i) \delta x_i| \le \Vert P \Vert \sum_{i = 1} ^ n \delta x_i \rightarrow 0\)

例:\(D(x) = \begin{cases}1&\mathbb Q \cap[0, 1] \\ 0 & [0, 1] \backslash \mathbb Q\end{cases}\)

可积函数性质

定义:\([a, b]\) 上可积函数的全体记称 \(\mathcal{R}[a, b]\)

定理:\([a, b]\) 上的连续函数或者单调函数一定可积(Rienman可积)

定理:(Lebesgue 定理)

\(f \in \mathcal{R}[a, b] \Leftrightarrow f\)\([a, b]\) 上几乎处处连续

积分的性质可以类比之前的东西

Newton-Leibniz 公式

定理:设 \(f \in \mathcal{R}[a, b]\),存在 \([a, b]\) 连续且 \((a, b)\) 可微函数 \(F(x), \text{s.t.} F'(x) = f(x)\)\(\int^b_a f(x) \mathrm{d}x - F(b) - F(a) - F(x) \vert_a ^ b\)

\[\begin{aligned} F(b) - F(a) &= \sum_{i = 1} F(x_i) - F(x_{i - 1}) \\ &= F'(\xi_i) \delta x_i\\ &=\sum_{i = 1} ^ n f(\xi_i) \delta x_i \end{aligned} \]

\(\Vert P \Vert \rightarrow 0\),那么 \(\int ^b _a f(a) = \mathrm{d} x = F(b) - F(a)\)

可积函数的性质

  1. 有界性,设 \(f\in \mathcal{R}[a, b]\)\(f\)\([a, b]\) 上有界

这里应该补一下

  1. 线性性

定理:设 \(f, g \in \mathcal{R}[a, b]\)\(\int \alpha f + \beta g \mathrm{d}x= \alpha \int ^ b _a f(x) \mathrm{d}x + \beta \int ^ b _a g(x) \mathrm{d}x\)

  1. 区间可加性

定理:设 \(f\in \mathcal R[a, b], c \in (a, b)\)\(f \vert_{[a, c]} \in \mathcal R[a, c], f\vert_{[c, b]}\mathcal{R}[c, b]\)

规定:

\(\int ^ b_a f(x) \mathrm{d}x = -\int ^ a_b f(x) \mathrm{d}x\)

  1. 绝对可以积性

定理:设 \(f\in \mathcal{R}[a, b], \vert f(x) \vert \in \mathcal{R}[a, b]\)\(\int _a ^ b f(x) \mathrm{d}x \le \int _a ^ b \vert f(x) \vert \mathrm{d}x\)

  1. 保序性

定理:设 \(f, g \in \mathcal{R}[a, b]\)\(f(x) \le g(x)\)\(\int ^ b _a f(x) \mathrm{d} x\le int ^ b_a g(x) \mathrm{d}x\)

  1. 积分中值定理

积分第一中值定理:

定理:设 \(f\in C([a, b])\)\(\exists \xi \in [a, b],\text{s.t.} \int ^ b _a f(x) \mathrm{d} x = f(\xi) (b - a)\)

带权积分中值定理:

\(f \in C([a, b]), g \in \mathcal{R}[a, b]\)\(g\)\([a, b]\) 上不变号,则 \(\xi \in [a, b], \text{s.t.} \int ^ b_a f(x) g(x) \mathrm{d}x = f(\xi) \int ^ b _a g(x) \mathrm{d}x\)

积分第二中值定理:

\(f, g\in \mathcal{R}[a, b]\), \(g\)\([a, b]\) 上单调,则 \(\exists \xi \in [a, b], \text{s.t.}, \int ^ b _a f(x) g(x) \mathrm{d}x = g(a) \int ^ \xi _a f(x) \mathrm{d}x + g(b) \int ^ b_\xi f(x) \mathrm{d}x\)

微积分基本定理

定理

定理:设 \(f\in \mathcal R[a, b]\),定义变上限积分 \(F(x) =\int ^ x_a f(t) \mathrm{d}t\),则由定义有 \(F(x) \in C([a, b])\)

定理2:设 \(f\in \mathcal R[a, b], x_0 \in [a, b]\)\(f\)\(x_0\) 点连续,则 \(F(x) = \int ^ x _a f(t) \mathrm{d}t\)\(x_0\) 处可导,并且 \(F'(x_0) = f(x_0)\)

定理3(微积分基本定理)

\(f\in C([a, b])\),则 \(f\)\([a, b]\) 上有原函数 \(F(x) = \int ^ x _a f(t) \mathrm{d}t\)\(\frac{\mathrm{d}}{\mathrm{d}t}\int ^ x_a f(t) \mathrm{d}t = f(x)\)

注:

  1. \(f\) 有有限多个间断点,则 \(F\)\([a, b]\) 上连续,并且除有限个点外,\(\frac{\mathrm{d}}{\mathrm{d}x} \int ^ x _a f(t) \mathrm{d}t = f(x)\)
  2. 除去有限个点外,\(F'(x) = f(x)\),称 \(F(x)\)\(f(x)\) 的广义原函数,
  3. \(f\) 有限多个间断点,则 \(NL\) 仍然成立

例子

例一:\(\psi:[\alpha,\beta] \rightarrow[a, b]\) 可微,\(f:[a, b] \rightarrow \mathbb R\) 连续,定义 \(F(t) = \int ^ {\psi(t)}_a f(x) \mathrm{d}x, G(t) = \int ^ b_\psi(t) f(x) \mathrm{d}x\),则 \(F'(t) = f(\psi(t))\psi'(t), G'(t) = -f(\psi(t))\psi'(t)\)

例二:计算 \(\lim\limits_{x\rightarrow \infty} \frac{\int ^ x_0 t ^ 2 e ^ {t ^ 2}}{x e ^ {x ^ 2}} = \frac{1}{2}\)

例三:设 \(f\in C([a, b])\) 且单调递增,则 \(\int ^ b_a x f(x) \mathrm{d}x \ge \frac{a + b}{2} \int ^ b_a f(x) \mathrm{d}x\)

分部积分和换元公式

分部积分

定理,设 \(u, v\in C ^ ([a, b])\)\(\int ^ b_a u v'\mathrm{d}x = uv \vert ^ b _a - \int ^ b_a u'v\mathrm{d}x\),其实只需要 \(u', v' \in \mathcal{R}[a, b]\) 即可

例1:

\(\int ^ 2 _1 x\ln x \mathrm{d}x = \frac{x ^ 2 \ln x}{2} \vert_1 ^ 2 - \frac{1}{2} \int ^ 2 _1 {x} \mathrm{d}x\)

例2:\(I_n = \int ^ {\frac{\pi}{2}}_0 \cos ^ n x\mathrm{d}x = (n - 1) \int ^ {\frac{\pi}{2}}_{0}\sin ^ 2 x \cos ^ {n - 2} x \mathrm{d}x = (n - 1) I_{n - 2} - (n - 1) I_n\)

积分型余次的 Taylor 公式

定理:设 \(f \in C ^ {(n + 1)} ([a, x])\)\(f(x) = \sum\limits _{k = 0} ^ n \frac{f^{(k)}(a)}{k!}(x - a) ^ k + \frac{1}{n!} \int ^ x _a f^{(n + 1)}(t) (x - t) ^ n \mathrm{d}t\)

注:

\[\begin{aligned} &\int ^ x _a f ^ {(n + 1)}(t) (x - t ) ^ n \mathrm{d} t \\ &=\int ^ x _a f ^ {(n + 1)}(x - t) ^ k (x - t) ^ {n - k } \mathrm{d}t\\ &=f ^{(n + 1)} (\xi)(x - \xi) ^ k \int ^ x _a(x - t) ^ {n - k} \\ &= f ^ {(n + 1)}(\xi) (x - \xi) ^ k \frac{(x - a) ^ {n - k + 1}}{n - k + 1} \end{aligned} \]

上述公式在 \(k = 0\) 时是 Lagrange 余项,在 \(k = n\) 的时候是 Cauchy 余项

原来满足 Lagrange 余项或 Cauchy 余项的 Taylor 公式只需要 \(f ^ {(n)}\)\([a, b]\) 上连续并且在 \((a, b)\) 上可导

换元

\(F:[a, b] \rightarrow \mathbb R\) 可微

\(\phi:[\alpha, \beta] \rightarrow [a, b]\) 也可微

\(\frac{\mathrm{d}F\phi(t)}{ \mathrm{d}t} = F'(\phi(t)) \phi'(t)\)

其中 \(F\)\(f\) 的原函数

定理:设 \(f\in C([a, b]), \phi:[\alpha, \beta] \rightarrow [a, b]\) 可微并且 \(\phi' \in \mathcal{R}[\alpha, \beta], \phi(\alpha) = a, \phi(\beta) = b\)

则有 \(\int ^ b_a f(x) \mathrm{d}x = \int ^ \beta_\alpha f(\phi(t)) \phi'(t) \mathrm{d}t\)

注:若 \(f\in \mathcal{R}[a, b]\) 即设 \(\phi\in C ^\infty, f\circ \phi\) 也不一定是 Riemann 可积的

定理:设 \(\phi:[\alpha, \beta]\) 可微且严格单调,\(\forall f \in \mathcal{R}[a, b]\) 则有 \(f(\phi(t)), \phi'(t) \in \mathcal{R}[\alpha, \beta]\)\(\int ^ {\phi(\beta)}_{\phi(\alpha)} f(x) \mathrm{d}x = \int ^ \beta_\alpha f(\phi(t))\phi'(t) \mathrm{d} x\)

定理:设 \(\phi:[\alpha, \beta]\) 可微且严格单调,\(\forall f \in \mathcal{R}[a, b]\) 则有 \(f(\phi(t)) \phi'(t) \in \mathcal{R}[\alpha, \beta]\)\(\int ^ {\phi(\beta)}_{\phi(\alpha)} f(x) \mathrm{d}x = \int ^ \beta_\alpha f(\phi(t))\phi'(t) \mathrm{d} x\)

可积理论(Darbowx)

\(f\in \mathcal{R}[a, b] \Rightarrow f\) 有界 必要条件

充分条件是什么?

记号

  • \((P, \xi)\)\([a, b]\) 的一个带标记分割
  • \(\Delta x_i = x_i - x_{i - 1}\)
  • \(\Delta _i = [x_{i -1}, x_i]\)
  • \(\tilde{P}\)\(P\) 的加细分割,若 \(\tilde{P}\)\(P\) 的分割增加分割点得到的分割。
  • \(\Delta_{ij} = [x_{i, j-1},x_{i, j}]\)
  • \(\Delta x_{i,j}= x_{i, j} - x_{i, j - 1}\)
  • \(\Delta_i = \bigcup\limits_{j = 1} ^ {m_i} \Delta_{ij}\)
  • \(\Delta x_i = \sum_{j = 1} ^ {m_i} \Delta x_{i, j}\)
  • 振幅 \(E\subset \mathbb R\) ,\(f\) 为定义在 \(E\) 上的函数 \(\omega(f;E) = \sup_{x_1, x_2\in E}\vert f(x_1) - f(x_2) \vert\)
  • Riemann 求和 \(S(f, P, \xi) = S(P, \xi) = \sum_{i = 1} ^ n f(\xi_i) \Delta x_i\)

可积的充分条件

定理:设 \(f\)\([a, b]\) 有界,\(\forall \varepsilon > 0, \exists > 0, \text{s.t.}\) 对于任意 \([a, b]\) 带标记点分割 \((P, \xi)\) 满足 \(\Vert P\Vert < \delta\),均有 \(\sum \omega (f;\Delta_i) \Delta x_i < \varepsilon\)\(f\in \mathcal{R}[a, b]\)

问:如果 \(f\in \mathcal R[a, b], \forall \varepsilon > 0, \exists \delta > 0\) 对于任意 \((P, \xi), \Vert P \Vert < \delta\) 是否有 \(\sum \omega(f;\Delta _i) \Delta x_i < \varepsilon\)

推论1:设 \(f \i C([a, b])\)\(f\in \mathcal R[a, b]\)

推论2:若 \(f\) 有界在 \([a, b]\) 上除有限个间断点外连续 则 \(f\in \mathcal{R}[a, b]\)

推论3:设 \(f:[a, b]\rightarrow \mathbb R\) 单调函数,则 \(f\in \mathcal{R}[a, b]\)

darboux 可积

定义:设 \(f:[a, b]\)\rightarrow\mathbb R$ 函数,\((P,\xi)\) 为一个分割令 \(m_i = \inf_{x\in \Delta _i} f(x), M_i = \sup_{x\in \Delta x_i} f(x)\)

\[\underline{S}(f; P) = \sum\limits^n_{i = 1}m_i \Delta x_i\\ \overline{S}(f; P) = \sum\limits^n_{i = 1}m_i \Delta x_i \]

称上面那个是 Darboux 上和,下面那个是 Darboux 下和,

\[\underline{S}(f; P) \le S(f; P, \xi) \le \overline{S}(f, P) \]

定理:

Darboux 上下和仅依靠分割,不依赖标记点。

加细之后,下和不减,上和不增

定义,Darbowx 上下积分

\[\underline{I} = \sup_{P} \underline{S}(f; P)\\ \overline{I} = \inf_{P} \overline{S}(f; P) \]

称上面的是 Darboux 下积分,记 \(\underline{\int} ^ b_a f(x) \mathrm{d}x\),称上面的是 Darboux 上积分,记 \(\overline{\int} ^ b_a f(x) \mathrm{d}x\)

定理 Darboux:设 \(f:[a, b]\rightarrow \mathbb R\) 有界函数

\[\underline{I} = \lim\limits_{\Vert P\Vert\rightarrow 0} \underline{S} (f; P) \\ \overline{I} = \lim\limits_{\Vert P\Vert\rightarrow 0} \overline{S} (f; P) \]

\(f:[a, b] \rightarrow \mathbb R\) 有界,那么 \(f\in \mathcal{R}[a, b] \Leftrightarrow \overline{I} = \underline{I} = I\)

只需要证明 \(\underline{I} = \lim\limits _{\Vert P \Vert \rightarrow 0} \underline{S}(f; P)\)

我们要证明 \(\forall \varepsilon > 0, \exists\delta > 0,\text{s.t.} \Vert P \Vert < \delta\) 时,

\[\underline{I} - \varepsilon < \underline{S}(f; P) \le \underline{I} \]

则存在分割 \(P_0\) 使得 \(\underline{I} - \frac{\varepsilon}{2} < \underline{S}(f; P_0)\),使得我们记 \(P_0\)\(k\) 个分割点,那么对于 \([a, b]\) 的任意分割 \(P\) 满足:

\[\underline{S}(f; P_0 \cup P) - \underline{S} (f; P) \le k \omega(f; [a, b])\Vert P\Vert \]

因此我们得到

\[\underline{S}(f; P) \ge \underline{S}(P \cup P_0) - k \omega M \Vert P \Vert > I - \frac{\varepsilon}{2} - k M \Vert P \Vert \]

则,在 \(\Vert P \Vert \rightarrow 0\) 时,\(\underline{I} \ge \underline{S}(f; P) > I - \varepsilon\)

如此,我们得到, \(\underline{I} = \underline{S}(f; P)\)

推论:(Darboux) \(f:[a, b] \rightarrow \mathbb R\) 有界函数,那么 \(\forall P, \lim \limits_{ \Vert P \Vert \rightarrow 0} \omega(f_i, \Delta_i) \Delta x_i = 0\)

Lebesgue 定理

零测集

\(\forall \varepsilon > 0, \exists \{I_\lambda\}_{\lambda \in \Lambda},\text{s.t.} E \subset \bigcup\limits_{i \in \Lambda} I_i, \sum\limits_{i \in \Lambda} \vert I_i\vert < \varepsilon\)

引理:\(f:E\rightarrow \mathbb R\) 函数

\(f\)\(x\in E\) 上连续 \(\Leftrightarrow \forall \varepsilon > 0, \exists \delta > 0, \omega(f; U_{E} (x; \delta)) < \varepsilon\)

Lebesgue 定理

\(f:[a, b]\) 上有界

\(f\in \mathcal{ R}[a, b]\Leftrightarrow f\)\([a, b]\) 上几乎处处连续,\(\text{i.e.}, f\)\([a, b]\) 上去掉一个零测集上连续。

可积函数性质

  1. \(f, g\in \mathcal{R}[a, b], \forall \alpha, \beta \in \mathbb R, \alpha f + \beta g \in \mathcal{R}[a, b]\)
  2. \(f \in \mathcal{R}[a, b]\)\(\vert f \vert \in \mathcal{R}[a, b], D(\vert f \vert) \subset D(f)\)
  3. \(f\in \mathcal{R}[a, b], \forall [\alpha, \beta] \subset [a, b]\)\(\left. f\right\vert_{[\alpha,\beta]}\in \mathcal{R}[\alpha,\beta]\)
  4. \(f, g \in \mathcal{R}[a, b]\)\(fg\in \mathcal{R}[a, b]\)

积分第二中值定理

定理:设 \(f, g\in \mathcal{R}[a, b], g\)\([a, b]\) 上单调,

\(\exists \xi \in [a, b], \text{s.t.} \int _a ^ b f(x) g(x) \mathrm{d}x = g(a) \int _a ^ \xi f(x) \mathrm{d}x + g(b) \int ^ b_\xi f(x) \mathrm{d}x\)

注:若 \(f, g\in C^{1}[a, b]\)\(F(x) = \int _a ^ x f(t) \mathrm{d}t,\int ^ b_af(x) g(x) \mathrm{d}x = \int _a ^ b g \mathrm{d} F =F g \vert_a ^ b - \int _a ^ b F g'\mathrm{d}x = g(b) \int _a ^ b f(x)\mathrm{d}x - \int ^b_a F(x)g'(x) \mathrm{d}x\)

由于第一积分中值定理 \(g'(x)\)\([a, b]\) 上不变号,所以 \(\exists \xi\) 使得

\[F(\xi)[g(b) - g(a)] = \int ^ b_a F(x) g'(x) \mathrm{d}x \]

带入回去就做完了。这是百度百科的做法。

Riemann-Lebesgue 引理:

\(f\in \mathcal{R}[a, b]\)

\[\lim_{\lambda\rightarrow +\infty}\int ^b_a f(x) \cos{\lambda}x = \lim_{\lambda\rightarrow +\infty}\int ^b_a f(x) \sin{\lambda}x = 0 \]

广义积分

广义积分

考虑在 \((0, 1]\) 上的连续函数 \(\frac{1}{\sqrt{x}}\) 其图像和 \(x = 0, x = 1\) 以及 \(x\) 轴的区域虽然无界,但是其面积是有界的。

定义:

设函数 \(f:[a, +\infty) \rightarrow \mathbb R\),在任意闭区间 \([a, b]\) 上可积,如果极限

\[\lim_{b\rightarrow + \infty} \int ^ b_a f(x) \mathrm{d}x \]

存在,那么称此为函数 \(f\)\([a, +\infty)\) 上的反常积分,记作

\[\int _a ^ {+\infty} f(x) \mathrm{d}x \]

注:如果极限不存在,那么也使用此符号,并称之为 \([a, +\infty)\) 上的反常积分,且发散的。如果极限存在,那么是收敛的。

我们可以类似的定义反常积分

\[\int _{-\infty} ^ a f(x) \mathrm{d}x \]

及其收敛性。

例 1.3:

考虑反常积分

\[\int_1 ^ {+\infty} \frac{\mathrm{d}x}{ x ^ p}, p\in \mathbb R \]

  • \(p > 1\) 时,积分收敛
  • 否则发散

定义:

设函数 \(f:[a, \beta) \rightarrow \mathbb R\) 在任一闭区间 \([a, b] \subset [a, \beta)\) 上可积, 如果极限

\[\lim_{b\rightarrow \beta^-} \int _a ^ b f(x) \mathrm{d}x \]

存在,则称瑕积分 \(\int _a ^ \beta f(x) \mathrm{d}x\) 收敛, 反之则称其为发散的。 这里 \(\beta\) 称为此反常积分的瑕点。

假定 \(f\)\((\alpha, b]\) 上有定义,在任一区间上 \([a, b] \subset (\alpha, b]\) 上可积,同样可以定义反常积分

\[\int ^ b_\alpha f(x) \mathrm{d}x \]

的敛散性.

例 1.4. 积分

\[\int ^ 1_0 \frac{\mathrm{d}x}{ x ^ p} \]

解: 因

\[\int ^ 1_a \frac{\mathrm{d}x}{ x ^ p} = \begin{cases} \left.\frac{x ^ {1 - p}}{1 - p} \right\vert _a ^ 1 & p \ne 1\\ \left.\log x\right \vert_a ^ 1& p = 1 \end{cases} \]

于是

  • \(p<1\) 时积分收敛
  • \(p\ge 1\) 时积分发散

注:

如果

  • 函数 \(f:[a, \beta)\rightarrow \mathbb R\) 在任一闭区间 \([a, b] \subset [a, \beta)\) 可积
  • \(\beta\) 的某一邻域 \((\beta - \delta, \beta)\) 上有界

则可以补充 \(f\)\(\beta\) 上的取值使得 \(f\) 成为 \([a, \beta]\) 上的有界函数。

由 Lebesgue 定理,\(f\)\([a, \beta)\) 上可积

\[F(x) = \int _a ^x f(t) \mathrm{d}t \]

连续,则有

\[\lim_{b\rightarrow \beta} \int _a ^ b f(x) \mathrm{d}x = \int _a ^ \beta f(x) \mathrm{d}x \]

上式右边的积分为通常意义下的 Riemann 积分。

统一记号:

\(\omega = \beta\)\(\omega =+\infty\) 上面考虑的反常积分通常记作

\[\int _a ^ \omega f(x) \mathrm{d}x \]

\(\omega = \alpha\)\(\omega =-\infty\) 上面考虑的反常积分通常记作

\[\int _\omega ^ b f(x) \mathrm{d}x \]

也将反常积分

\[\int _b ^ \omega f(x) \mathrm{d}x = -\int ^b_\omega f(x) \mathrm{d}x \]

如果函数 \(f:[a, \omega) \cup (\omega, b]\rightarrow\mathbb R\) 在任意区间 \([a, c]\subset [a, \omega)\)\([d, b] \subset (\omega, b]\) 上可积,那么可以考虑反常积分

\[\int _a ^ \omega f(x) \mathrm{d}x, \int _\omega ^ b f(x) \mathrm{d}x \]

如果他们都收敛,那么反常积分

\[\int _a ^ b f(x) \mathrm{d}x \]

也收敛,反之如果有两个中有一个是发散的,那么其也是发散的。

反常积分的性质和通常意义下的积分类似。

分部积分

\(f, g \in C ^ 1([a, \omega))\)

  • \(f\cdot g', f'\cdot g\)\([a, \omega)\) 的反常积分以及极限
    \(\left.f(x) g(x) \right\vert_a ^ \omega = \lim\limits_{x\rightarrow \omega ^-}f(x) g(x) - f(a) g(a)\)

中两者收敛,那么则有

\[\int_a ^ \omega f(x) \mathrm{d}g(x) = \left.f(x) g(x)\right\vert _a ^ \omega - \int _a ^ \omega g(x) \mathrm{d} f(x) \]

\(\text{NL}\) 公式:

设在 \(f:[a, \omega)\rightarrow \mathbb R\) 在任意区间 \([a, b]\subset[a, \omega)\) 上可积,在 \([a, \omega)\) 上有原函数 \(F\)

\(\int_a ^ \omega f(x) \mathrm{d}x\) 收敛当且仅当 \(\lim\limits_{x\rightarrow \omega ^ -} F(x) = F(\omega)\) 存在,此时有 \(\int_a ^ \omega f(x) \mathrm{d}x = F(\omega) - F(a)\)

Cauchy 判别法

设在 \(f:[a, \omega)\rightarrow \mathbb R\) 在任意区间 \([a, b]\subset[a, \omega)\) 上可积,则 \(\int_a ^ \omega f(x) \mathrm{d}x\) 收敛当且仅当 \(\forall \varepsilon > 0, \exists b\in [a, \omega)\) 使得对于 \(\forall b_1, b_2 \in (b, \omega)\)

\[\left\vert \int ^ {b_2}_{b_1} f(x) \mathrm{d}x\right\vert<\varepsilon \]

如果

\[\int_a ^ \omega \vert f(x) \vert \]

收敛,那么称这个反常积分是绝对收敛的

如果一个反常积分绝对收敛,那么他肯定收敛。

设非负 \(f:[a, \omega)\rightarrow \mathbb R\) 在任意区间 \([a, b]\subset[a, \omega)\) 上可积,则 \(\int_a ^ \omega f(x) \mathrm{d}x\) 收敛当且仅当函数

\[F(x) = \int _a ^ x f(t) \mathrm{d}t \]

有界。

推论:

\(f\)\([1, +\infty)\) 的非负单调递减函数,那么无穷级数 \(\sum_{i = 1} ^ \infty f(i)\) 和反常积分 \(\int _a ^ \omega f(x) \mathrm{d}x\) 同收敛

比较判断法:设函数 \(f, g:[a, \omega)\rightarrow \mathbb R\) 在任意区间 \([a, b] \subset [a, \omega)\) 均可积,且 \(0\le f(x) \le g(x),\forall x \in [a, \omega)\),则当 \(\int _a ^ \omega g(x) \mathrm{d} x\) 收敛时,\(\int _a ^ \omega f(x) \mathrm{d}x\) 也收敛。

反之,如果 \(\int _a ^ \omega f(x) \mathrm{d}x\) 发散,那么 \(\int _a ^ \omega g(x) \mathrm{d}x\) 也发散。

推论:设函数 \(f, g:[a, \omega)\rightarrow \mathbb R\) 在任意区间 \([a, b] \subset [a, \omega)\) 均可积,且 \(0\le c_1f(x) \le g(x)\le c_2 f(x),\forall x \in [a, \omega)\),则 \(f, g\) 的反常积分同收敛

条件收敛

如果一个反常积分收敛但是不绝对收敛,那么就是条件收敛的。

\(f, g:[a, \omega)\rightarrow \mathbb R\) 在任意区间 \([a, b]\subset[a, \omega)\) 上可积且 \(g\)\([a, \omega)\) 上单调,那么

  • Abel 判别法:\(\int _a ^ \omega f(x) \mathrm{d}x\) 收敛,并且 \(g\)\([a, \omega)\) 有界
  • Dirichlet 判别法:\(F(x) = \int_a ^ x f(t) \mathrm{d}t\)\([a, \omega)\) 上有界,且 \(\lim\limits_{x\rightarrow \omega^-} g(x) = 0\)

之一成立,则

\[\int _a ^ \omega f(x) g(x) \mathrm{d}x \]

收敛。

定义:两端的反常积分

\[\int ^ {\omega_2} _{\omega_1} f(x) \mathrm{d}x = \int ^ {\omega_2} _{c} f(x) \mathrm{d}x+\int ^ {c} _{\omega_1} f(x) \mathrm{d}x \]

如果上式右段端的两个反常积分都收敛,那么左边也收敛。

\(\text{Beta}\) 函数:

\[\int _0 ^ 1 x ^ {p - 1} (1 - x) ^ {q - 1} \mathrm{d}x \]

这个东西在 \(p > 0, q > 0\)

\(\text{Gamma}\) 函数:

考虑函数

\[\Gamma(x) = \int _0 ^ {+\infty} t ^ {x - 1} e ^ {-t} \mathrm{d}t \]

\(x > 0\) 时有定义

posted @ 2024-01-17 09:21  siriehn_nx  阅读(162)  评论(3编辑  收藏  举报